Download as docx, pdf, or txt
Download as docx, pdf, or txt
You are on page 1of 41

LEGALEDGE TEST SERIES

Part of the Most Awesome and Consistently Successful Study Material and Test Series Module, spanning across
both Physical and Online Programs in the entire Country. While most of the world fumbled and faltered, 2020
has been another inspiring Success story both for us and those who chose to trust us. As a result LE was able to
engineer Clean-Sweep-Landslide figures of a handsome 35 Selections under 100 ranks, and a whopping 180
selections under 500 ranks in CLAT 2020. With AILET being no different, a total of 30 LEtians found their way
into NLUD in 2020. Read on!

MOCK COMMON LAW ADMISSION TEST 2021


MOCK CLAT #24

Candidate Name : Batch


Duration : 120 Minutes
:
Max. Marks : 150
Contact No. :
Centre Name :
Date of Exam :

INSTRUCTIONS TO CANDIDATES

1. No clarification on the question paper can be sought. Answer the questions as they are.
2. There are 150multiple choice objective type questions.
3. Each question carries ONE mark. Total marks are 150.
4. There is a negative marking of 0.25 marks for every incorrect answer.
5. Candidates have to indicate the correct answer by darkening one of the four responses provided, with a
BALL PEN (BLUE OR BLACK) in the OMR Answer Sheet.
Example: For the question, "Where is the TajMahal located?", the correct answer is (b). The
candidate has to darken the corresponding circle as indicated below :
(a) Kolkata (b) Agra (c) Bhopal (d) Delhi
Right Method Wrong Methods

6. Answering the questions by any method other than the method indicated above shall be considered incorrect
and no marks will be awarded for the same.
7. More than one response to a question shall be counted as wrong.
8. The candidate shall not write anything on the OMR Answer Sheet other than the details required and in the
spaces provided for.
9. After the Test is over, the candidate has to return the OMR Answer Sheet to the invigilator. The
candidate should take the Test Paper along with them.
10. The use of any unfair means by any candidate shall result in the cancellation of his/her candidature.
11. Impersonation is an offence and the candidate, apart from disqualification, may have to face criminal
prosecution.
12. Electronic gadgets like mobile phones, pagers or calculators are strictly not permitted inside the Test
Centre/Hall.
13. The candidates shall not leave the hall before the Test is over.
SECTION-A : ENGLISH LANGUAGE

Directions(Q.1-Q.29): Read the passages carefully and answer the questions.

Passage (Q.1-Q.5): The announcement of dates for Bengal, Tamil Nadu, Kerala, Assam and Puducherry
assembly elections has focused attention on the strong repercussions their outcome will have for national politics.
BJP governs Assam, is a strong challenger in Bengal, and its ally AIADMK administers TN. Any setback for BJP in
Bengal and Assam and ally AIADMK in TN could bring the opposition roaring back into contention in national
politics. Consequently, BJP will face greater questioning over farm policy, high fuel prices and attempts to pursue
economic reforms like privatisation. But a BJP/ NDA victory in these three states will pose existential questions to
its rivals. For instance, a strong opposition voice like Mamata Banerjee losing sway over Bengal will signal a
version of pan-India primacy of BJP.
Not surprisingly, the contestation is fiercest in Bengal, with both sides boasting evenly matched resources.
Foreseeing intense political violence, Election Commission has announced an eight-phased poll for Bengal, which
some fear may only prolong the unrest. With Mamata invoking Bengali pride to counter the rush of Hindi heartland
leaders campaigning against her, BJP is making a strong pitch for “poriborton” and attempting to ride anti-
incumbency in Bengal.
Much like 2019, PM Modi remains BJP’s most saleable face in Bengal. But an assembly election may dim Modi’s
traction, requiring local leaders to up their game. With Assam and Bengal elections coinciding, the CAA has forced
BJP into a delicate balancing act. While the party hopes CAA will give it huge traction among East Bengal Hindu
migrants, it’s unpopular in Assam where BJP has to underplay it. Delays in notifying CAA rules can be read as a
symptom of BJP’s dilemma, allowing TMC to dub it a gimmick.
Pursuing a third straight term in TN, AIADMK passed a quota for the influential Vanniyar community. AIADMK
must also grapple with Sasikala’s return from prison, which could turn into a full blown power struggle if the party
loses. TN would worry BJP for its sizeable industrial base, which will bolster UPA’s fundraising ability if it wins.
Wiped out of Bengal and Tripura, CPM faces a difficult task in Kerala with its history of voting out incumbents. For
Congress, wins in Kerala, Assam, TN and Puducherry will help regain lost respectability amid constant setbacks
and counter growing dissidence against Rahul Gandhi. The summer of 2021 promises big changes in national
politics.

1. What is the passage primarily about?


(a) the fact that Assembly elections in four states could reconfigure national politics
(b) the interrelated political alliances of various parties
(c) the controversies regarding CAA and BJP’s maneuvering it in its favor
(d) the notion of political parties devising policies not for the betterment of their citizens but to stay in power

2. These passage mentions the following:


(i) Bengal (ii) Tamil Nadu
(iii) Kerala (iv) Assam
(v) Puducherry
Which of the above are currently governed by the BJP, as per the passage?
(a) All of the above
(b) (i), (ii) and (iv)
(c) (i), (ii), (iii)
(d) only (i)

3. Which of the following is a direct consequence of BJP losing power in any of the above mentioned States?
(a) the influence of Mamta will become paramount in Bengal, making her undefeatable in the subsequent elections
(b) BJP might get a lot of slack for their national policies and governance
(c) BJP might have problems passing policies
(d) might lead to Rahul Gandhi’s political redemption
4. Which of the following is untrue about the impending elections in Bengal?
(a) both the parties are almost at par in the State, and the results can tilt either way
(b) the people of Bengal are likely to vote for BJP because of Modi, but mostly because of the popularity of the
local BJP leaders in Bengal, which is the main worrying point for Mamta
(c) Both the parties have gotten down to using community based politics
(d) all of the above are true

5. Which of the following is true about the CAA?


(a) CAA was passed by the BJP government
(b) CAA is widely popular in all the states mentioned above
(c) The delay in notifying the CAA rules might lead to TMC winning the elections
(d) all of the above

Passage (Q.6-Q.9): On Thursday, government introduced sweeping new rules that encompass a wide spectrum of
digital content. But the way in which these rules force-fit social media, streaming entertainment, and digital news
portals all under one umbrella, is untenable. News content already undertakes compliance with various standalone
legislations. Plus news sites follow print and TV norms, besides the extensive self-regulation done in multiple layers
between journalists and editors every day. Social media by contrast has run [X], without accountability.
Leave alone discussing these far-reaching changes with stakeholders in digital news, government has sidestepped
even a cursory parliamentary scrutiny by introducing the sweeping regulatory framework as subordinate legislation.
And now, at a time when news media are already battling adverse economic and legal environments, with serious
charges like sedition being levelled on frivolous grounds, a whole new set of compliance measures will bring fresh
costs and dangers. For example, the oversight authority set up with government officials and suo motu powers could
both encourage strong-arming by the state and trolls mounting a deluge on a selected media target, to hurt it
punitively.
Government’s aim to impose accountability on social media to tackle a pandemic of fake news and hatred is
sound. But it’s the mainstream news platforms that have offered the strongest checks on this dangerous
phenomenon, by amplifying the credible facts and information. The three-tier grievance redressal mechanism
proposed in the rules is only going to put constraints on this work. It is not minimum government, maximum
governance and it’ll not provide benefits to society. It’s best to drop the current rules. Instead, government can use
the ongoing parliamentary exercise in fleshing out a data protection legislation to meet its objectives of regulating
social media content.

6. Which of the following is the underlying theme of the passage?


(a) news platforms must not be clubbed with social media
(b) the new rules must be scraped as they are regressive
(c) the new rules do not serve the purpose the government is trying to solve
(d) the government must unbridle news media outlets

7. How is news content different from the other types of content it is clubbed with under the new rules?
(a) it goes through various check points, unlike other forms of content
(b) it is more important and relevant than the other forms of content
(c) it is more nation centric than the other forms of content
(d) all of the above

8. What can effectively replace [X] in the above passage?


(a) independent
(b) infamous
(c) rogue
(d) amuck
9. These new rules have comes as a result of which of the following?
(a) the almost negligible accountability of social media sites
(b) to counter the ever prevalent false information easily available on the internet
(c) the regulate the digital content, which has far reaching effects on the public
(d) to curb sedition against the government

Passage (Q.10-Q.14): Migrants move in search of safer and better lives. They contribute to the welfare and
sustainable development of their countries of origin and destination. Yet, they face complex human rights
challenges and vulnerabilities that we must address to ensure that no one is left behind.
In the Asia-Pacific region, the number of international migrants has grown from 52 million in 1990 to over 65
million today, roughly 25% of all the world’s international migrants. Notably, 70% of all international migrants in
Asia-Pacific come from within the region.
Most of the region’s migrants send remittances to families and others in their origin countries. This is important
because remittances support household consumption and contribute to poverty reduction. Between 2009 and 2019,
remittances to the region rose from $183 billion to $330 billion, nearly half of the 2019 global total of $717
billion. But since COVID-19, remittances have declined drastically. Those to Eastern Europe and Central Asia
declined over 16% from $57 billion in 2019 to $48 billion in 2020. Remittances in East Asia and the Pacific have
fallen more than 10% over the same period, from $147 billion to $131 billion.
The Asia-Pacific Migration Report 2020 shows that voluntary and involuntary causes drive migration between
countries in Asia and the Pacific and in other regions of the world. The primary reason is temporary labour
migration. Many people also migrate for education, to escape poverty and inequality, food insecurity and climate
change, to reunite with family or for permanent settlement and retirement. People often move for more than one
reason.
Migrants often lack access to essential services, constrained by laws, fees, language barriers and restrictions related
to residency and migration status. Women migrants, especially domestic workers, are particularly at risk of
discrimination, violence, abuse and exploitation. Migration-related child protection risks are also a significant
concern throughout Asia and the Pacific.

10. The main theme of the passage revolves around:


(a) In the Asia-Pacific region, the number of international migrants has grown since 1990.
(b) Safe and Orderly Migration in the Asia-Pacific is needed.
(c) Basic statistical scenario of international Migration before and after pandemic.
(d) Rights and exploitation of international migrants throughout Asia and the Pacific.

11. Which of the following option best represent the meaning of the word ‘remittance’ as has been mentioned in the
context of the passage?
(a) a sum of money sent in payment or as a gift.
(b) the use of energy, time, or other resources.
(c) a payment, especially one made by a solicitor to a third party and then claimed back from the client.
(d) the action or process of getting rid of something.

12. Which of the following statements is/are true?


Statement 1: In Eastern Europe and Central Asia, remittances have rose drastically since COVID-19. Statement 2:
Women migrants, especially domestic workers, are particularly at risk of discrimination, violence, abuse and
exploitation.
(a) Statement 1
(b) Statement 2
(c) Statement 1 and 2
(d) Neither Statement 1 nor 2
13. According to the passage, which of the following is the main cause that drive migration between countries in Asia
and the Pacific and in other regions of the world?
(a) Education
(b) Food Insecurity and Climate change
(c) Temporary labour migration
(d) To escape poverty and insecurity

14. Which of the following option best represent the meaning of the word ‘involuntary’ as has been mentioned in the
context of the passage?
(a) denoting an outstanding or supreme example of a particular kind of person or thing.
(b) causing one to lose courage or confidence; disconcerting
(c) done against someone's will; compulsory.
(d) (especially of a position or view) not able to be maintained or defended against attack or objection.

Passage (Q.15-Q.19): In the early 1900s when legendary musicologist, Pandit V.N. Bhatkhande, reached the
south of India as part of his epic sojourn through the country to consolidate music systems, he was disappointed.
In accounts of these tours recorded in his travelogue, Majha Dakshinecha Pravaas (My Musical Journey in Southern
India), and in some of his letters and lectures, he expressed his dissatisfaction not only with what he described as
“this yaiyy yaiyy style of singing” but also with the fact that south of India did not quite turn out to be the land of
untouched Shastric music that he was seeking. It was instead a colourful amalgam that included folk, Islamic and
Christian musical styles too. The ardent, anti-Muslim Sanskritist in him was not pleased.
Bhatkhande was not alone in his conception and more than a hundred years after his not-so-pleasant discovery, this
narrative of the ‘pure South’ continues to drive the scholarship and public consumption of its classical arts.
According to Davesh Soneji, Associate Professor, Department of South Asia Studies, University of Pennsylvania,
this notion has resulted in the erasure of a rich tradition of Tamil Muslim and Christian music and many other art
forms practised by non-Brahmin or Hindu artist communities.
At a webinar titled ‘The Arduous Arts: Caste, History, and the Politics of “Classical” Dance and Music in South
India’, organised by the University of California, Los Angeles, on January 11, Soneji and others examined these and
related issues that rarely get discussed. With conversations about equity gaining prominence, said Professor Anna
Morcom, Mohindar Brar Sambhi Chair of Indian Music, UCLA Department of Music, “now seemed the right time
to run this panel, because of the momentum of BLM which is causing more discussion and reflection about class
amongst South Asian scholars internationally and in India… The debates on caste are becoming more open and
heated especially in the world of south Indian classical dance… (and) it is imperative that these debates are
grounded in evidence-based research… I was really keen to present a panel on caste with a highly authoritative
lineup of leading scholars, performers, and activists,” she said.
The theme of erasure – a natural corollary of hegemony – resonated persistently throughout its 3.5-hour duration.
T M Krishna, singer and activist who has persistently talked about this issue, raised the question of ‘aesthetic
erasure’, a process by which [musical] sounds that do not conform to the classical Carnatic style are deemed
plebian and thus undeserving of recognition. It begs the question of what indeed is “classical” and who decides its
parameters.

15. Which of the following statements is/are true with respect to the context of the passage?
Statement1: Pandit V.N. Bhatkhande, was amused when he reached the south of India as part of his epic sojourn
through the country to consolidate music systems.
Statement 2: The accounts of the tours recorded in his travelogue is called, Majha Dakshinecha Pravaas.
(a) Statement 1
(b) Statement 2
(c) Statement 1 and 2
(d) Neither Statement 1 nor 2
16. Which of the following options best convey the meaning of the word ‘hegemony’ as has been used in the passage?
(a) leadership or dominance, especially by one state or social group over others.
(b) the practice or quality of including or involving people from a range of different social and ethnic
backgrounds and of different genders, sexual orientations, etc.
(c) (in Hinduism and Buddhism) the sum of a person's actions in this and previous states of existence, viewed as
deciding their fate in future existences.
(d) praise and honour received for an achievement.

17. The main theme of the passage revolves around:


(a) a brief account of a recent webinar organized by the UCLA Center for India and South Asia, where scholars and
artists laid bare the seldom-discussed problem of caste in the performing arts of South India.
(b) The life and work of legendary musicologist, Pandit V.N. Bhatkhande.
(c) A discussion on ‘aesthetic erasure’, a process by which [musical] sounds that do not conform to the classical
Carnatic style are deemed plebian and thus undeserving of recognition.
(d) detailed discussion on the erasure of a rich tradition of Tamil Muslim and Christian music and many other art
forms practised by non-Brahmin or Hindu artist communities.

18. Which of the following best represent the meaning of the word ‘plebian’ as has been used in the passage?
(a) member of the general citizenry in ancient Rome as opposed to the privileged patrician class (mainly used as a
derogatory term for pupils of lower social classes).
(b) the practice of engaging in the same behavior or activity for which one criticizes another or the practice
of claiming to have moral standards or beliefs to which one's own behavior does not conform.
(c) a strong supporter of a party, cause, or person.
(d) a lack of cooperation between rival political parties.

19. Given below are the effects of the narrative of the ‘pure South’ which continues to drive the scholarship and
public consumption of its classical arts. Choose the option which contains the correct answer.
1) Professor Anna Morcom, Mohindar Brar Sambhi Chair of Indian Music, UCLA Department of Music, got
disappointed when she visited South India and experienced the cultural environment there.
2) The debates on caste became more open and heated especially in the world of south Indian classical dance.
3) South of India did not quite turn out to be the land of untouched Shastric music.
4) Erasure of a rich tradition of Tamil Muslim and Christian music and many other art forms practised by non-
Brahmin or Hindu artist communities.
Codes:
(a) (2), (3) and (4)
(b) only (1)
(c) (1) and (3)
(d) only (4)

DONE TILL HERE


Passage (Q.20-Q.24): In December 1943, a 20-year-old named Ruth Elias arrived in a cattle car at the
Auschwitz-Birkenau concentration camp. She was assigned to Block 6 in the family camp, a barracks that housed
young women and the camp’s male orchestra, an ensemble of incarcerated violinists, clarinet players, accordion
players and percussionists who played their instruments not just when the prisoners marched out for daily labor
details, but also during prisoner floggings.
Performances could be impromptu, ordered at the whims of the SS, the paramilitary guard of the Nazi Party. In a
postwar interview, Elias discussed how drunken SS troops would often burst into the barracks late at night.
Music is often thought of as inherently good, a view exemplified in the playwright Wilhelm Congreve’s oft- cited
aphorism “music hath charms to soothe a savage beast.” It is also often seen as a form of art that ennobles those
who play and listen to it. Its aesthetic qualities seem to transcend the mundane and horrific.
Yet it’s also been used to facilitate torture and punishment, a topic I think is worth exploring. When I was
researching my book Drunk on Genocide: Alcohol and Mass Murder in Nazi Germany, I was struck by the ways in
which music accompanied deaths in the camps, the ghettos and the killing fields.
The fusion of alcohol, music and song with mass murder shows how violence was normalised – even celebrated –
by the Nazis. Under the Nazi regime, music and song forged community, camaraderie and shared purpose. In unit
bars, around campfires and at the killing sites, the addition of music was more than just a form of entertainment. It
was also an instrument for promoting a common purpose and bringing people together. Through rituals of song,
drink and dance, the Nazis’ actions could be collectivised and normalised – and their larger project of violence that
much easier to pull off.
Ultimately, genocide is a societal endeavour; music and song – like political philosophies – are part of a society’s
cultural artefacts. So when mass murder becomes a central tenet of a society, perhaps it shouldn’t be a surprise that
these atrocities are carried out against a backdrop of stirring song, a rousing military march or a sentimental
Schumann melody.

20. Which of the following option correctly represents the meaning of the word ‘tenet’ as has been mentioned in the
passage?
(a) a principle or belief, especially one of the main principles of a religion or philosophy.
(b) tending to keep a firm hold of something; clinging or adhering closely.
(c) able to be maintained or defended against attack or objection.
(d) very weak or slight.

21. Which of the following statement/s is/are true according to the context of the passage?
Statement 1: The book Drunk on Genocide: Alcohol and Mass Murder in Nazi Germany, was written by Wilhelm
Congreve.
Statement 2: Music was an instrument for promoting a common purpose and bringing people together in the Nazi
regime.
(a) Statement 1
(b) Statement 2
(c) Statement 1 and 2
(d) Neither statement 1 nor 2

22. Which of the following option correctly represents the meaning of the word ‘camaraderie’ as has been used in
the passage?
(a) a small group of people, especially a group of advisers to a ruler or politician, with a shared purpose.
(b) a warning or proviso of specific stipulations, conditions, or limitations.
(c) a dashing and attentive man, especially one acting as a lady's escort.
(d) mutual trust and friendship among people who spend a lot of time together.

23. According to the passage, which one of the following was the main purpose of music during the Nazi regime?
(a) Playing music helped them gain popularity among the masses.
(b) playing music was just a form of entertainment around campfires.
(c) music was used to validate and normalize genocide as a purpose and part of a society’s cultural artefacts.
(d) for their enjoyment as they held drunken parties.

24. Which of the following writing styles does the author follow in this passage?
(a) Narrative
(b) Persuasive
(c) Expository
(d) Descriptive
Passage (Q.25-Q.29): Under the auspices of the Corbett Centre for Maritime Policy Studies comes a thoughtful
book by Jeremy Stocker, Architects of Continental Sea-power: Comparing Tirpitz and Gorschkov, shedding light
on Grand Admiral Tirpitz and Admiral Sergei Gorschkov, two important naval figures of the 20th century, who
wrestled with the problems and challenges of creating formidable navies for primarily continental countries.
Tirpitz was a product of Wilhelmine Germany and used his contacts with chancellor Bulow to satisfy Kaiser
Wilhelm’s ambition to make a ‘great power’ of the newly created state. The Kaiser’s passion for heraldry, uniforms,
military pomp and ceremony carried itself forward into a disastrous foreign policy, which would eventually lead to
war and the destruction of the nascent state.
How much of an accomplice in this tragic endeavour was Tirpitz is rightly brought out by the author, who writes
that Tirpitz’s ambition need not necessarily have resulted in a fleet of battleships, but battleships were large, with an
overwhelming presence and glory. This, unlike, for instance, the U-boats of Germany that eventually did more
damage to the enemy in the first world war.
The author rightly criticises both the Kaiser and Tirpitz for having unsatisfactory strategic objectives, in building a
battle line, other than giving Germany ‘a place in the sun’ and building a navy of a ‘great power’. Having served in
the navy at about the time that Alfred Thayer Mahan was writing his great work, The Influence of Sea Power Upon
History: 1660–1783, Tirpitz did not seem to have taken Mahan’s caution of maritime geography, seriously enough.
Although given enough funds to build three capital ships a year, the High Seas Fleet played little part in influencing
the land-centric centre of gravity, permitting as many as seven infantry divisions to be landed by Britain on the
coast of France, without interference.
Tirpitz’s ambition set off a naval arms race with Britain, in which the latter, with its vast financial resources, easily
outpaced the German building capacity. Eventually, Britain, the chief trading partner of pre-war Germany, opted for
a strategy of a distant blockade, which crushed the morale of the German people and forced the yet undefeated
German army to sue for peace.
Tirpitz was partly right in foreseeing that building a battle fleet would result in a Mahanian decisive battle, and so it
occurred at Jutland, but it was too little, too late. The British Grand Fleet though tactically defeated, continued to
rule the waves, while Admiral Hipper was forced to shelter Wilhelms. The author rightly says that if the funds spent
on Tirpitz’s fleet had been given to the army, the German army might have pulled it off on the Western front.

25. The main theme of the passage revolves around which of the following options?
(a) Discussing and comparing two important naval figures of the 20th century, who wrestled with the
problems and challenges of creating formidable navies for primarily continental countries.
(b) Discussing modern naval tactics and warfare strategies.
(c) Discussing about how the British Grand Fleet got defeated but continued to rule the waves, while Admiral
Hipper was forced to shelter Wilhelms.
(d) Discussion about how the funds spent on Tirpitz’s fleet should have been spent more effectively and wisely.

26. Which of the following options correctly conveys the meaning of the word ‘nascent’ as has been used in the
passage?
(a) having or showing an excessive interest in or admiration of oneself and one's physical appearance.
(b) a condition characterized by an extreme tendency to fall asleep whenever in relaxing surroundings.
(c) (especially of a process or organization) just coming into existence and beginning to display signs of future
potential.
(d) (of a person or manner) feeling or appearing casually calm and relaxed; not displaying anxiety, interest,
or enthusiasm.
27. Which of the following statement/s is/are true as per the context of the passage?
Statement 1: The Influence of Sea Power Upon History: 1660–1783, was written by Alfred Thayer Mahan.
Statement 2: The Kaiser’s passion for heraldry, uniforms, military pomp and ceremony carried itself forward
into a disastrous foreign policy.
(a) Statement 1
(b) Statement 2
(c) Statement 1and 2
(d) Neither Statement 1 nor 2.

28. Which of the following options correctly represent the meaning of the word ‘infantry’ as has been used in the
passage?
(a) soldiers marching or fighting on foot; foot soldiers collectively.
(b) incapable of making mistakes or being wrong.
(c) a general increase in prices and fall in the purchasing value of money.
(d) an arrival or entry of large numbers of people or things.

29. Which of the following reasons crushed the morale of the German people and forced the yet undefeated German
army to sue for peace?
(a) The U-boats of Germany eventually did more damage to the enemy in the first world war.
(b) Tirpitz’s ambition set off a naval arms race with Britain, in which the latter, with its vast financial resources,
easily outpaced the German building capacity and Britain being the chief trading partner of pre-war Germany,
opted for a strategy of a distant blockade.
(c) The Kaiser’s passion for heraldry, uniforms, military pomp and ceremony carried itself forward into a
disastrous foreign policy.
(d) As a product of Wilhelmine Germany, Tirpitz used his contacts with chancellor Bulow to satisfy Kaiser
Wilhelm.
SECTION-B : GENERAL KNOWLEDGE/CURRENT AFFAIRS

Directions (Q.30–Q.66): Read the information given below and answer the questions based on it.

Passage (Q.30-Q.34): Starlink is a plan by [1] to put [2] satellites into low Earth orbit (LEO) that offer high-
speed, low-latency, cheap internet access to anyone anywhere on the planet. That’s the end-game. All you would
need to use Starlink is a $200 pizza box-sized receiver. Each satellite will talk to four others using lasers as they
constantly orbit Earth, together creating a web of Ku-band and Ka-band broadband connectivity as fast as the speed
of light that surrounds the planet at all times, and for all locations.
In order to beam connectivity to the surface, a massive network of ground-based stations will also be necessary. So
although [2] satellites sounds like a lot, it's only a fraction of the infrastructure that SpaceX will have to construct.
Space internet is simply satellite-powered Internet access. This is not a new thing. Telecommunications satellites
mostly sit in a geostationary orbit thousands of miles above Earth's equator and follow the direction of Earth's
rotation, so appearing to stay in one place to serve one region. Their distance from Earth means a lag of about a
second or more.

30. Which of the following will replace [1] in the above passage?
(a) Tesla
(b) Space X
(c) Blue Origin
(d) Virgin Galactic

31. Low Earth Orbit extend up to how many km above Earth’s surface?
(a) 2000 km
(b) 1000 km
(c) 500 km
(d) 200 km

32. Which of the following is one of the advantages of the Space internet?
(a) The signals from satellites in space can overcome obstacles faced by fibre-optic cables
(b) The presence of satellite at a lower height from the Earth’s surface, will help to bring the lag down to 20-30
milliseconds.
(c) Space Internet will be able to deliver the internet in remote areas or places with difficult terrain.
(d) All of the above.

33. What is the name of Space Internet project launched by Amazon?


(a) Blue Origin
(b) OneWeb
(c) Project Kuiper
(d) Virgin Galactic

34. Which of the following will replace [2] in the above passage? (a)
4000
(b) 8000
(c) 10000
(d) 12000

Passage (Q.35-Q.39): A SpaceX Falcon 9 rocket launched a new batch of 60 Starlink internet satellites into orbit
early Sunday (March 14) and nailed its landing at sea to top off a record-setting mission. The flight comes just days
after the company's last launch, where a different Falcon 9 rocket took off from neighboring Space Launch
Complex 40, at Cape Canaveral Space Force Station. Both missions delivered a full stack of 60 Starlink satellites to
orbit, helping SpaceX inch ever-closer to filling its initial constellation of 1,440 satellites.
With an eye to further strengthen India’s friendly ties with Bangladesh, Prime Minister Narendra Modi Tuesday
virtually inaugurated the 1.9-km-long ‘Maitri Setu’ or the India-Bangladesh Friendship Bridge, built over [1] that
connects Sabroom district in south Tripura to Ramgarh in Bangladesh.
Inaugurating the bridge, Modi said it will not only boost friendly ties between the two neighbours but also prove to
be a strong link of business. Bangladesh Prime Minister [2] in a video message said the bridge will usher a new
chapter in relations between the two countries.
Announced by Prime Minister Narendra Modi in 2015, the 1.9 km-double lane bridge, which also includes approach
roads, was constructed at a cost of Rs 133 crore by the [3], a government-owned company under the road transport
and highways ministry.

35. Which of the following will replace [1] in the above passage?
(a) Kushiyara River
(b) Karnaphuli River
(c) Brahmaputra River
(d) Feni River

36. The 1.9 Km long bridge joins in Tripura with in Bangladesh.


(a) Sabroom and Ramgarh
(b) Ramgarh and Sabroom
(c) Belonia and Matiranga
(d) Matiranga and Belonia

37. Which of the following will replace [2] in the above passage?
(a) Khaleda Zia
(b) Sheikh Saima Wazed
(c) Sheikh Hasina
(d) Sheikh Rehana

38. Which of the following will replace [3] in the above passage?
(a) National Highways and Infrastructure Development Corporation Limited
(b) National Highway Authority of India
(c) Ministry of Road Transport
(d) Border Roads Organisation

39. The Maitri Setu is being called which of the following?


(a) Gateway of Northeast
(b) Gateway of Bangladesh
(c) Gateway of friendship
(d) Gateway of India

Passage (Q.40-Q.44): The World Bank report released Tuesday said an analysis done by them previously
found that improving transport connectivity between the two countries could result in a 172 per cent increase in
India’s exports to Bangladesh and a 297 per cent increase in Bangladesh’s exports to India.
Along with the Maitri bridge, Modi also laid the foundation stone of an Integrated Check Post (ICP) at
Sabroom. The ICP aims to ensure seamless movement of goods and passengers between India and Bangladesh.
The leaders of the "Quad" group of nations - a regional alliance seen as part of efforts to balance China's growing
military and economic power - discussed "free, open and inclusive Indo-Pacific region" as well as the coronavirus
pandemic.
The leaders of the "Quad" group of nations - a regional alliance seen as part of efforts to balance China's growing
military and economic power - discussed "free, open and inclusive Indo-Pacific region" as well as the coronavirus
pandemic.
Prime Minister Narendra Modi said, "It is good to be among friends. I thank President Biden for this initiative. We
are united by our democratic values and our commitment to a free, open and inclusive Indo-
Pacific. Our agenda today covers areas like vaccines, climate change and emerging technologies make the Quad a
force for global good."
"I see this positive vision as an extension of India's ancient philosophy of Vasudeiva Kutumbakam which regards
the world as one family. We will work together, closer than ever before for advancing our shared values and
promoting a secure, stable and prosperous Indo-Pacific," he said.
"Today's summit meeting shows that Quad has come of age. It will now remain an important pillar of stability in the
region," added. President Biden said a free and open Indo-Pacific region was essential to the futures of all four
countries.
The Biden administration has said the Quad nations will announce financing agreements to support an increase in
manufacturing capacity for coronavirus vaccines in India, something New Delhi has called for to counter China's
widening vaccine diplomacy.

40. Which of these countries is not the part of Quadrilateral framework?


(a) Japan
(b) Australia
(c) Russia
(d) None of these

41. Which of these was not the major topic of discussion in the summit?
(a) areas of the cooperation to maintain a free, open and inclusive Indo-Pacific region
(b) regional and global issues which are of shared interest
(c) efforts to combat the Covid-19 pandemic
(d) Skirmishes with China

42. QUAD is also known as the Asian


(a) NATO
(b) SCO
(c) APEC
(d) UIJA Summit

43. This forum was initiated by Shinzo Abe with the support of Prime Minister of India, Manmohan Singh; Vice
President of US, Dick Cheney in which year?
(a) 2007
(b) 2008
(c) 2010
(d) 2011

44. USA sees this grouping as an opportunity to regain its influence in the Indo-Pacific region in a counter to
increasing influence in the East Asia by
(a) Iran
(b) Russia
(c) China
(d) UAE

Passage (Q.45-Q.49): At the back of nationwide protests against government policies and a crackdown on civil
liberties, India has fallen two places in the 2020 Democracy Index report released by The Economist Intelligence
Unit (EIU).
A significant and novel aspect of this year remains that the Coronavirus pandemic’s impact on freedom and
democracy was also measured. Asia’s regional score fell to its lowest since 2013, as policies adopted by
governments to curb the spread of the virus in these countries resulted in stringent constraints on individual
freedom and civil liberties.
The country’s ranking over the period has seen a sharp drop from the 27th position in 2014 with a score of 7.92.
The report alleges the political fabric of the country has been damaged by religious strife and anti-Muslim feeling
under the Bharatiya Janata Party (BJP) who it claims has purposely created policies that have heightened discontent.
45. How many countries are covered under this report?
(a) 165
(b) 166
(c) 167
(d) 168
46. Which of these is not among the categories in which report is classified?
(a) Flawed
(b) Hybrid
(c) Authoritarian
(d) Laisse-Faire

47. How many countries falls under the category of ‘full democracies’?
(a) 23
(b) 32
(c) 53
(d) 42

48. Which one of these neighbouring countries don’t fall in the category of Hybrid Regime?
(a) Pakistan
(b) Bhutan
(c) Bangladesh
(d) Sri Lanka

49. What was India’s rank in the year 2019?


(a) 50th
(b) 51st
(c) 52nd
(d) 53rd

Passage (Q.50-Q.54): While presenting the Union budget for the FY 2021-22, Finance Minister Nirmala
Sitharaman announced that Operation Green (OG) will be expanded beyond tomatoes, onions, and potatoes (TOP)
to 22 perishable commodities. Although we don’t know yet which other commodities have been included in OG, we
welcome this move as it reflects the government’s intentions of creating more efficient value chains for perishables.
Out of the Rs 500 crore from its initial outlay, Rs 50 crore were reserved for the price stabilisation objective,
wherein [1] was to intervene in the market wherever prices crashed due to a glut, to procure some excess arrivals
from the surplus regions to store them near major consuming centres. Another Rs 450 crore has been reserved for
developing integrated value chains projects. Such projects are given 50 per cent grants-in-aid with a maximum
limit of Rs 50 crore per project. This subsidy goes up to 70 per cent in case the project is of a Farmer Producer
Organisation (FPO). As of February 23, six projects worth Rs 363.3 crore have been approved for the scheme, of
which Rs 136.82 crore has been approved as grant-in-aid. But so far, a mere Rs. 8.45 crore has been actually
released, which may be because the scheme envisages the payment of subsidy on a reimbursement basis.

50. Which of the following has been replaced by [1] in the above passage?
(a) NAFED
(b) AIFPA
(c) IFCA
(d) FSSAI

51. Operation Green was originally launched in (a)


2017
(b) 2018
(c) 2019
(d) 2020
52. There were three basic objectives when OG was launched. Which of these is not one of them?
(a) containing the wide price volatility
(b) building efficient value chains
(c) reducing the post-harvest losses
(d) increase of domestic agricultural production and sector contribution to the Gross Domestic Product (GDP)

53. What is/are issue(s) in Operation Green replicating operation flood?


(a) There are so many varieties of TOP vegetables, grown in different climatic conditions and in different seasons,
making marketing intervention (processing and storage) all the more complex.
(b) TOP are mostly traded in APMC markets, with layers of mandi fees and commissions, and farmers get less
than one-third of the consumer’s rupee.
(c) Both a and b
(d) None of these

54. For increasing higher Processing-to-Production Share, which of the following strategies is not relevant?
(a) the government should run campaigns in association with industry organizations, as was done for eggs.
(b) the government needs to promote the food processing units in horticulture
(c) distributing milk through an organized retail network.
(d) There has to be a separate board to strategize and implement the scheme

Passage (Q.54-Q.58): Over the past days and weeks, some instances of multilateral diplomatic activism have been
witnessed towards eliminating the deadlock that has been formed around the issue of the United States’ potential return to
the JCPOA. Vienna, Geneva, Brussels, Paris, Berlin, Washington, New York, and Tehran have been playing host to this
activism.
U.S. President Joe Biden’s attempts to revive the Joint Comprehensive Plan of Action (JCPOA), better known as the Iran
nuclear deal, have not seen any breakthrough with both sides waiting for the other to blink. The Biden administration says
it would return to the deal if Iran starts complying with its terms. Tehran, on the other side, asks the U.S., which
unilaterally quit the deal under the Donald Trump administration in May 2018, to return to the agreement first and lift
sanctions on Iran. The EU’s efforts to organise direct U.S.-Iran talks were also unsuccessful as Tehran reportedly rejected
the offer. Iran has also accelerated its nuclear programme. This game of chicken continues as the clock is ticking. Iran will
elect a new President in June. Hassan Rouhani, who bet his presidency on the deal — only to be repudiated by Mr.
Trump — cannot stand in a third consecutive election. There is no guarantee that a moderate like Mr. Rouhani would be
elected this time. And it is not a secret that there is considerable opposition among the hardliners, a powerful
constituency, towards any kind of engagement with the U.S. Mr. Biden’s best bet is to get the nuclear agreement
back on track before Mr. Rouhani leaves office.
To be sure, Mr. Biden has moved with a sense of urgency after assuming power. He appointed a special envoy for Iran,
showed signs of rebalancing ties with Saudi Arabia, and sent clear signals to Tehran about America’s desire to get back to
the deal. But these actions do not seem to be enough to rebuild the trust after the acrimonious Trump years. Some of Iran’s
concerns are genuine. After the September 11, 2001 terrorist attack, Iran had cooperated with the U.S. in the war against
the Taliban. But once the Taliban were driven out of power, the Bush administration branded Iran as part of an “Axis of
Evil” along with Iraq and North Korea. As President Barack Obama offered diplomacy, the Iranians grabbed the
opportunity, leading to the signing of the JCPOA in 2015. And Iran was fully compliant with the agreement when Mr.
Trump pulled the U.S. out of it. So Iran would seek some consistency in U.S. policy.
55. Joe Biden has been sworn as the president of the United States
(a) 45th
(b) 44th
(c) 46th
(d) 47th

56. J--CPOA was signed in 2015, but former US President Trump has withdrawn from it in (a)
2016
(b) 2017
(c) 2018
(d) 2019

57. The JCPOA was the result of prolonged negotiations from 2013 and 2015 between Iran and P5+1. Which one of the
following is not the part of it?
(a) China
(b) France
(c) Germany
(d) India

58. The U.S. decision of withdrawing was criticized by all other parties to the JCPOA (including the European allies)
because Iran was in compliance with its obligations, as certified by the
(a) International Atomic Energy Agency
(b) European Organization for Nuclear Research
(c) Institute of Nuclear Materials Management
(d) International Energy Agency

Passage (Q.59-Q.62): The National Cyber Security Policy, which was first drafted in the wake of reports that the
US government was spying on India and there were no technical or legal safeguards against it.
India was among the first few countries to launch a cybersecurity policy back in 2013 and now it is looking to
revamp that framework. According to Bloomberg, the new rules will be approved by the cabinet committee on
security headed by Prime Minister Narendra Modi. Considering the fluid nature of cyberattacks, the law should
enable collaborative efforts amongst states and union territories of the country and also at a cross-jurisdiction level,"
explained Supratim Chakraborty, a partner at Khaitan & Co.
More importantly, experts are hoping that this time the regulations will have a little more bite in order to be
effective against cyber threats. “India's cyber threat canvas has evolved and now includes persistent and penetrating
attacks targeting India's national and commercial computer networks. The move to set up a new system comes
amid increasing reports of hackers from other countries targeting users in India and the country’s critical
infrastructure. According to cybersecurity firm Recorded Future, a new group called the [1] has its cyber guns
trained on India’s power grid, and may be linked to the massive power outage in Mumbai that crippled banks and
the glitch at the National Stock Exchange (NSE), which the authorities are still investigating.

59. Which of the following has been replaced by [1] in the above passage?
(a) Bureau 121
(b) Morpho
(c) Red Echo
(d) APT28 (Fancy Bear)

60. According to EY’s latest Global Information Security Survey (GISS) 2018-19, the highest number of cyber threats
have been detected in which country?
(a) USA
(b) UK
(c) India
(d) China
61. According to which section of the Information Technology Act, CII is defined as a “computer resource, the
incapacitation or destruction of which, shall have debilitating impact on national security, economy, public health or
safety”?
(a) Section 70(1)
(b) Section 71(4)
(c) Section 69(2)
(d) Section 56(3)

62. Which agency is specialized within the United Nations and plays a leading role in the standardization and
development of telecommunications and cyber security issues?
(a) Internet Corporation for Assigned Names and Numbers (ICANN)
(b) Internet Governance Forum (IGF)
(c) International Telecommunication Union (ITU)
(d) None of these

Passage (Q.63-Q.66): The government under the ambit of the [1], brought in detailed guidelines for digital
content on both digital media and Over The Top (OTT) platforms, while giving overriding powers to the
government to step in.
The new rules were unveiled at a joint press conference by Information Technology Minister Ravi Shankar Prasad
and Information and Broadcasting Minister Prakash Javadekar. The Centre's new, stricter guidelines for social
media intermediaries make it mandatory for platforms such as WhatsApp to aid in identifying the “originator” of
“unlawful” messages.
It also requires the likes of Twitter, Facebook and YouTube to take down such messages within a specific time-
frame, set up grievance redressal mechanisms as well as assist government agencies in investigation. The guidelines
have invited criticism that instead of soft-touch monitoring, the government has opted for predatory new rules.

63. The new guidelines on social media were framed in accordance with which section of the Information
Technology Act, 2000
(a) section 78 (2)
(b) section 72 (1)
(c) section 88 (1)
(d) section 87 (2)

64. Which IT Act provides the Central Government with the authority to issue instructions to public, to restrict online
access to information
(a) Section 67
(b) Section 67A
(c) Section 69
(d) Section 69A

65. Under which sections of the IT Act, Safe Harbour Provision is defined?
(a) Section 69
(b) Section 59
(c) Section 79
(d) Section 89

66. Which of the following will replace [1] in the above passage?
(a) Information Technology (Intermediary Guidelines and Digital Media Ethics Code) Rules 2021.
(b) Information Technology (Intermediary Guidelines) Rules 2021
(c) Information Technology (Intermediary Guidelines on Social Media and OTT Platforms) Rules 2021
(d) Information Technology (Intermediaries Restrictions) Rules 2021
SECTION – C :LEGAL REASONING

Directions (Q.67 – Q.104): Read the comprehensions carefully and answer the questions based on it.

Passage (Q.67-Q.71): The Calcutta High Court recently had occasion to rule that even a trespasser, unless
evicted by the due process of law, is entitled to electricity (Sukla Kar v. The Calcutta Electric Supply
Corporation Ltd. & Ors.)
Justice Arindam Mukherjee, however, clarified that even the electricity connection is granted to such a person, it
would not create any other right in her favour as regards the property.
The petitioner in this case had moved the High Court after her application for a new service connection was
rejected by the Calcutta Electric Supply Corporation (CESC). Mere possession doesn’t mean that the
possessor will have any rights, title and interest in respect of the said premises.
The CESC told the Court that it cannot provide a new service main in the premises occupied by the petitioner as
there already existed once. However, the petitioner can be provided a new meter and a new connection from the
existing main service, the Court was informed.
Other private respondents told the Court that the petitioner did not have rights over the property where the
electricity connection was sought. It was submitted that she had earlier been enjoying electricity from an existing
meter on the basis of an arrangement between the petitioner and the private respondents. On account of failure to
pay bills, the electricity was disconnection, they contended.
The High Court opined that the disputes over the premises, whether it be ownership, title or interest, cannot stand in
the way of the petitioner getting a new electricity meter in her name, when the possession of the premises by her is
admitted.
"Assuming without admitting that the petitioner does not have any right, title and interest in respect of the said
premises, the petitioner’s status then is that of a trespasser. Even a trespasser, unless evicted by due process of law,
is entitled to electricity. Electricity connection, if granted to the petitioner, will not also create any right in her
favour", reads the order.
Therefore, Justice Mukherjee proceeded to direct the petitioner to make a new application for a meter and new
electricity connection within 7 days. It further issued directions for the CESC to process the application, inspect the
premises, raise a quotation and install a new meter to grant the petitioner a new electricity connection.
The Court added that it has not gone into the dispute between the petitioner and the private respondents, nor will its
order create or affect any right for the petitioner in respect of the premises.
[Meera Emmanuel, ‘Even a trespasser, unless evicted by law, is entitled to electricity: Calcutta High Court’ <
https://www.barandbench.com/news/litigation/even-a-trespasser-unless-evicted-by-law-is-entitle- to-electricity-
calcutta-high-court> as accessed 27th December, 2020]

67. A has been in possession of B’s house since long. C, his neighbour files a case against him on the grounds
that his cow ate leaves from a poisonous tree from his garden and died. Decide.
(a) A is not liable because it was not A who planted the tree
(b) is not liable since he just had a mere possession of the house and not the garden
(c) is not liable because he did not do anything intentionally
(d) is not liable because having possession doesn’t mean he has rights and liabilities for the same

68. A being the possessor of B’s house even after lawful eviction had stopped paying the electricity bills after which he
requested for a new connection in his name for which he was rejected. He had therefore filed a case regarding the
same. Decide
(a) He will lose since he has no rights over the property
(b) He will lose since he is just a mere possessor of the property
(c) He has to clear the previous bills; only then can he get a new one
(d) None of the above
69. A being the possessor of B’s house had stopped paying the electricity bills after which he requested for a new
connection in his name for which he was rejected. He had therefore filed a case regarding the same.
Decide
(a) He will lose since he has no rights over the property
(b) He will lose since he is just a mere possessor of the property
(c) He has to clear the previous bills; only then can he get a new one
(d) None of the above

70. A was staying at B’s property for past 3 years. The property had a huge garden in which many beautiful shrubs
including poisonous ones grew. Once, it so happened that A’s dog chew on a such a poisonous flower and died. A
then sues B for the same. Will he win?
(a) No, because B is no longer liable for his property.
(b) Yes, because it is B’s property.
(c) No, because A should have been careful while walking his dog.
(d) Yes, because B should’ve let A know of the types of plants present in the garden.

71. A has possession of B’s property. A had stopped paying electricity bills and thus his electricity was cut off. He
therefore went on to file an application for a new connection on July 24 th, 2020 for which she was denied one. On
July 25th, 2020, he got an eviction notice from the court. She files a petition regarding the same. Decide
(a) She will get the connection since had asked for one before she was served an eviction notice.
(b) She will not be given the connection since currently she is no more entitled to electricity.
(c) She will not get the electricity connection since has to first clear the previous bills
(d) She will not get the electricity connection since the whole country is under lockdown and it is
impossible to get a connection during the lockdown period

Passage (Q.72-Q.76): ‘Indians have nothing to fear,’ has been the refrain of the proponents of the Citizenship
Amendment Act (CAA). On the corollary move to implement a National Register of Citizens (NRC), they keep
changing their positions — sometimes they say NRC and CAA have no link at all; other times they say the CAA
will protect non-Muslims excluded from the NRC exercise. At times they say it is an internal matter of India, but
they also promise to send back the “infiltrators” to places where they allegedly came from. The argument that
Indians have nothing to fear — made repeatedly by Prime Minister Narendra Modi and Home Minister Amit Shah
among others — is less a reassurance and more an assertion of the intent of the NRC-CAA. Yes, Indians have
nothing to fear. But to be assured of no fear, she has to first PROVE that she is an Indian! The NRC puts onerous
requirements that most Indians will find unable to meet, Hindu or Muslim. But non-Muslims have, at least
theoretically, a route to citizenship under CAA. Muslims, if they cannot get into NRC — as a large number of them
like all other communities, will end up — will have no route to citizenship.
But the statement that ‘Indians have nothing to fear’ — while there are many reasons why they are worried
— also suggests that those who are worried, the protesters, are not Indians or less Indians. The very act of protest
itself undermines one’s credentials as Indians. Considering the reflex response of the regime that labels every critic
of it ‘anti-national,’ this is a new twist. If you are Indian, you have nothing to worry. And inversely, if
you are worried, you may not be Indian![Source:
https://www.thehindu.com/news/national/opinion-citizenship-amendment-act-the-fear- factor/article80881479.ece]

72. Which of the following views can be correctly attributed to the author of the above passage?
(a) The Indians should not take to the streets to protest against the CAA NRC as the newly introduced law will not
affect them in any manner.
(b) The intention of the CAA NRC law is to make the Indians feel less worried about their citizenship
(c) Despite multiple assurances by Prime Minister Modi and Home Minister Amit Shah there is speculation that the
CAA NRC might strip off the citizenship of many people including Hindus as well. Hindus might be able to use
this to retain their citizenship but the Muslims will have no way out.
(d) Indians should not be worried about CAA NRC exercise as it is being brought about to deal with the Muslim
infiltrators in the country
73. As per the passage given above what is the reason given by the government for bringing the CAA law?
(a) To send back the Muslim infiltrators where they belong
(b) To protect the non-Muslim excluded from the NRC exercise
(c) To strip the citizenship of the protesters who are less Indian
(d) To strip the citizenship of the government critic and Anti National

74. Suppose if one Mr X is a Muslim and another Mr Y is a Hindu. Both X and Y feel to satisfy the requirement under
the NRC. As per as per the law explain above what effect will the CAA have on their status of citizenship?
(a) Both Mr X and Mr will be denied citizenship as the field in the NRC exercise
(b) Both Mr X and Mr Y will be granted citizenship as even though they were excluded by the NRC they will be
observed as citizens under the CAA
(c) Mr X will continue to be a citizen of India as per the provisions of the CAA but Mr Y will not be
considered to be a citizen of the country
(d) Mr Y will continue to be a citizen of India as per the provisions of the CAA but Mr X will not be
considered to be a citizen of the country

75. The author has expressed his views in support of the NRC CAA exercise. The above given statement is true or
false?
(a) True as the author constantly argues that the Indians have nothing to worry about the NRC CAA
(b) False, as the author has argue that the concept of who is an Indian has itself come into question after the
government's response to criticism
(c) True as the author appreciates the reassurance given by Prime Minister Modi and Home Minister Shah that
Indians have nothing to worry about
(d) False as the author is worried about the non-secular nature of the law

76. According to the above passage what is the response of the government to every critic?
(a) Critics will have no route to citizenship under the NRC CA
(b) Critics are labeled as Anti National and less Indian and thus they are raising their voice against the law
(c) Critics should not worry as they are Indians and the law will not affect Indians
(d) Critics will not be able to meet the onerous requirements and the NRC

Passage (Q.77-Q.82): A contract of insurance is very similar to indemnity contracts. Here, the insurer promises
to compensate the insured for his losses. In return, he receives consideration in the form of premium. However, the
Contract Act does not strictly govern these kinds of transactions. This is because the Insurance Act and other such
laws contain specific provisions for insurance contracts.
Parties under Indemnity Contracts
There are generally two parties in indemnity contracts. The person who promises to indemnify for a loss is the
Indemnifier. On the other hand, the person whose losses the indemnifier promises to make good is the Indemnified.
We can also refer to the Indemnified party as the Indemnity Holder. For example, in the earlier example, C is the
Indemnifier and B is the Indemnity Holder.
Nature of Indemnity Contracts
An indemnity contract may be either express or implied. In other words, parties may expressly create such a
contract as per their own terms. The nature of circumstances may also create indemnity obligations impliedly. For
example, A does an act at the request of B. If B suffers some losses and A offers to compensate him, they impliedly
create an indemnity contract.
Rights of an Indemnity Holder
When parties expressly make a contract of indemnity, they can determine their own terms and conditions. However,
sometimes they may not do so. In such a case, the indemnity holder can enforce the following rights against the
indemnifier:
1) The indemnifier will have to pay damages which the indemnity holder will claim in a suit.
2) The indemnity holder can even compel the indemnifier to pay the costs he incurs in litigating the suit.
3) If the parties agree to legally compromise the suit, the indemnifier has to pay the compromise amount.
77. A wheelchair manufacturer enters into an agreement with a large hospital to provide 500 wheelchairs at a discount
price. The manufacturer asks that a clause be included in the contract, in which the hospital agrees to protect the
company from any losses or lawsuits should patients be injured while using any of the wheelchairs. Is the following
an indemnity clause?
(a) Yes, it is, as the patients would be compensated incase they face any injury.
(b) No, injured patients would be indemnified, and since they are not party to the contract this won’t be an
indemnity clause.
(c) Yes, as the hospital indemnifies the wheelchair company, or the hospital guarantees for any losses or injuries
that may occur.
(d) No, as the manufacturer is not protected from any litigations or compensatory charges.
78. Lola has a homeowner’s insurance policy on her home in Texas. The insurance company has agreed to indemnify
Lola against damages to her home and the personal property kept there, from many types of damage, including fire,
burglary, and liability if someone gets injured on Lola’s property. A visiting neighbor trips on a crack in the
walkway and falls, breaking her arm. Will her contract protect her from liability?
(a) No, as the indemnity is only in terms of injury from fire, burglary or someone getting hurt due to these.
(b) No, as the person tipped on the walkway which is not part of the home and outside it’s periphery.
(c) Yes, the insurance company would protect Lola from the medical bills and other losses claimed by the
neighbor by paying the claim.
(d) Yes, as the contract of indemnity protects you from all the future losses incurred to you.
79. Luke takes his car to the shop for repairs that will take a few days. The shop offers Luke a loaner car so he can get
back and forth to work. Luke signs the shop’s loaner car agreement, which requires Luke have insurance, and
includes an indemnifing clause. While driving through town in in the loaner car, Luke rear-ends a car at a stoplight.
Luke suffers minor injuries, but the driver of the other car has several moderate injuries, and the damage to the
loaner car is substantial. A couple of weeks later, the other driver demands payment from the repair shop, as owner
of the car that hit her, for medical bills, repairs to her own car, and pain and suffering. Who is responsible for this
compensation?
(a) Luke as his written agreement to indemnify the repair shop protects the business from liability.
(b) The lady will claim it from the repair shop and later they can make the dealing with Luke based on
compromise.
(c) Luke, who is solely responsible for any events surrounding the accident, as he was driving the car.
(d) The repair shop as they are the owner of the car and thus responsible for who they let drive their vehicle.

80. Robert is remodelling his house to transform it into the home of his dreams. The front porch and yard are being torn
down and reconstructed, leaving Robert to worry about whether one of the contractors may trip over debris and
sue him for injuries. To protect himself, Robert asks all of the contractors and laborers working on his home
to sign an indemnity and hold harmless agreement for the same. A worker while leaving the site for his house, falls
and injures himself over the pavement. Will he be indemnified by Robert?
(a) No, he won’t as the injury is not work related.
(b) Yes, he would, as he wouldn’t have been hurt if he wasn’t leaving from work itself.
(c) No, as the indemnity agreement protects him for paying for any liability caused due to injury.
(d) Yes, as he is the indemnifier for all liabilities caused to his workers employed on the site.

81. Mr. X is a surgeon and currently doing practice in India. Mr. Y offers him Job in Pakistan and asks him to start his
practice here. Mr. Y also promises him that he will save him from loss incurred due to practice here. Identify the
indemnifier?
(a) Mr X is the indemnifier because he benefits from the contract.
(b) Mr. Y is the indemnifier because he would take up the liability.
(c) Indemnity contracts cannot work in foreign nations, as it is part of Indian Contract Act.
(d) Third person would be indemnifier as, indemnity contracts require a third person to bear the liability.
82. A contracts with the Governments to return to India from abroad after completing his studies and serve the
Government for a fixed period. He fails to return to India. Is this a contract of indemnity?
(a) This is a contract of indemnity and he is bound to reimburse the Government.
(b) This is a contract of indemnity but he is not the indemnifier.
(c) This is not a contract of indemnity but a contractual obligation.
(d) This is not a contract of indemnity as it does not fulfill the requisites for it.

Passage (Q.83-Q.88): Exceptions to Offence of Murder under Section 300 IPC, which could mitigate it to
culpable homicide not amounting to murder are:-
Exception I – Grave and Sudden Provocation as mitigation
Culpable homicide is not murder if the offender, whilst deprived of the power of self-control by grave and sudden
provocation, causes the death of the person who gave the provocation or causes the death of any other person by
mistake or accident.
Essentials – The following conditions must be complied with in order to invoke the benefits of this clause:-
1. The deceased must have given provocation to the accused.
2. The provocation must be grave.
3. The provocation must be sudden.
4. The offender, by reason of the said provocation, should have been deprived of his power of self- control.
5. The accused killed the deceased during the continuance of the deprivation of the power of self- control.
6. The offender must have caused the death of the person who gave the provocation or that of any other person by
mistake or accident.
Exception II – Exceeding the Right of Private Defence
Culpable homicide is not murder if the offender, in the exercise in good faith of the right of private defence of
person or property, exceeds the power given to him by law and causes the death of the person against whom he is
exercising such right of defence without premeditation, and without any intention of doing more harm than is
necessary for the purpose of such defence.
Exception III – Public servants exceeding his powers
Culpable homicide is not murder if the offender, being a public servant or aiding a public servant acting for the
advancement of public justice, exceeds the powers given to him by law, and causes death by doing an act which he,
in good faith, believes to be lawful and necessary for the due discharge of his duty as such public servant and
without ill-will towards the person whose death is caused.
Exception IV — Sudden Fight
This exception applies to cases wherein death is caused in a sudden fight without premeditation in the heat of
passion in a sudden quarrel; so long as the fight is unpremeditated and sudden, the accused, irrespective of his
conduct before the quarrel, earns the mitigation provided for in Exception 4 to Section 300, IPC. Subject to the
condition that he did not in the course of fight take undue advantage of or act in a cruel or unusual manner.
Exception V – Consent
Culpable homicide is not murder when the person whose death is caused, being above the age of eighteen years,
suffers death or takes the risk of death with his own free and voluntary consent.
[Source: Indian Penal Code, 1860, https://www.legalbites.in/law-notes-ipc-exceptions-to-offence-of- murder/]

83. Ram came home from work and found that his neighbor Sunil was molesting his 8 years old daughter. He was so
shocked and angry that he took axe and went towards Sunil to kill him. Seeing him, Sunil pushed him and ran for
his life. Ram ran behind Sunil with the axe and found him 6 hours later hiding behind a bus. Ram caught him and
killed him with his axe. Decide whether the exception of Grave and sudden provocation will apply or not.
(a) Yes, because Sunil’s act could be considered as an heinous crime and was grave in nature.
(b) Yes, because seeing his daughter getting molested, the father was filled with rage and anger.
(c) No, because the act of killing Sunil was not sudden.
(d) Yes, because the act of Sunil deserves to be punished in this manner.
84. Raj was sleeping in his house when a thief entered his house. Due to the noise, Raj woke up and caught the thief.
The thief tried to threaten raj with a baseball bat kept there and started to escape from the house. Raj in order to
prevent the thief from escaping, took out his pistol from the shelf and fired at him, thus killing him. Will he be sued
for murder or any exception can be granted.
(a) No, because the thief was threatening raj and could have harmed it, therefore raj acted in private defence
(b) Yes, because killing is not a justified punishment for stealing.
(c) Yes, because Raj used more power than was necessarily required for private defence.
(d) No because the thief committed mistake and should have been punished.

85. X was a policeman who was trying to catch a dangerous armed culprit, who was hiding in a factory. Unaware of
this, the watchman of the factory who was taking round suddenly appeared in front of the policeman holding his
gun. As it was dark, “X” couldn’t see properly and shot him dead. Will he be charged with murder?
(a) Yes, because it was because of his negligence that the watchman got killed.
(b) No, because he fired in discharge of his duty, thinking him to be the armed culprit.
(c) No, because it was an accident which took place because it was dark.
(d) Yes, because the policeman should have given warning before firing bullet.

86. Mukesh and Nitin had a conflict due to which they had physically fought many times before. Mukesh wanted to end
this chapter so he called Nitin and asked to meet him in person. He carried with him a knife. Soon their
conversation got heated up and both got involved in a physical fight. Finding the right time, Mukesh stabbed nitin
with his knife many times. Will any of the exception apply here?
(a) Yes, the exception of grave and sudden provocation will be applicable as Mukesh killed Nitin in the heat of
the moment.
(b) No, because the exception of sudden fight cannot be applied as it was planned by Mukesh
beforehand and was executed in a cruel manner.
(c) No, because he killed Nitin out of personal conflict and rage.
(d) Yes, because he acted in self defence against Nitin.

87. A 19 year old blind person named Jai was living with his guardian who used to be very rude to him. He used to
torture Jai over his disability and taunted him everyday that he is a burden to everyone. Finally, Jai got depressed
and requested his guardian to kill him in order to give him relief from all the pain. Will the guardian be liable of
murder?
(a) Yes, because the consent given by Jai was not voluntary and free will, he was forced by his guardian.
(b) Yes, because he killed jai because of his personal motive.
(c) No, because the guardian relieved Jai from all the pain and sufferings.
(d) No because the guardian had the consent of Jai who was over the age of 18 and had given voluntary consent.

88. “A” is a police officer who was investigating a matter related to terrorism in Delhi. He arrested few people who
were suspected to be involved in the crime. During investigation, he found that none of them were involved in the
crime and had to release them. One of those suspect “X” met the police officer “A” while he was not on duty and
engaged into a physical conflict with him. “X” tried to stab “A” with a knife that he brought with him but “A”
quickly shot him dead with his personal licensed gun. Will “A” will be charged of murder.
(a) Yes, because he did not kill “X” in the discharge of his duty therefore the exception of public servant
exceeding his powers is not available.
(b) No, because he killed “X” out of private defence who was otherwise trying to stab him to death.
(c) No, because he killed “X” out of mistake.
(d) Yes, because he went to meet “X” with a personal gun which shows that it was pre planned.
Passage (Q.89-Q.94): In an interesting order, Justice Rajiv Narain Raina of the Punjab and Haryana High Court
adjourned a Civil Revision on Petitioner's counsel's pleading that the court was in a bad mood and he would like
to argue the case on some other day. The order dated February 4 discloses that Justice Raina dismissed four
urgent cases listed before him that day, one after the other. This led to the Petitioner's counsel, Advocate KS Sidhu,
assuming that the court was not in the right mood to allow his case.
"Counsel assuming that mood of the Court is bad this morning dismissing the first four urgent cases one
after the other with orders dictated in Court, prays that time may be granted to him to argue the case on
some other day. I grant permission for an adjournment but not without saying that those cases were not
worth admission," the order read.
The rules for adjournments are listed under Order XVII of the CPC. Rule 1 thereunder clearly stipulates that the
Court may grant an adjournment to a party if "sufficient cause" is shown; provided that no such adjournment
shall be granted more than three times to a party during hearing. Sub-Rule (2) of Rule 1 further stipulates that
adjournments are to be granted only if the circumstances are "beyond the control" of the party who seeks it.
However, these rules may be applicable only in cases of civil courts and not any other authorities. [Extracted
(edited) from: LiveLaw News Network, https://www.livelaw.in/news- updates/lawyer-seeks-adjournment-as-the-
mood-of-the-court-is-bad-hc-allows-the-request-read-order- 152972]

89. Anahad Narain was arguing before the NCDRC which is a commission defending Piramal Healthcare Limited in a
tough case. As the judge started grilling him on the nitty-gritties of the matter, Narain started to find it
uncomfortable as there were too many things he did not have answers to. Narain feigned unconsciousness. As he
pretended to regain his consciousness, he sought an adjournment citing ill- health from the Chairman of the
Commission. To his utter shock, the chairman denied such adjournment. Narain argued that adjournment must be
granted for the circumstances were beyond his control. Is the chairman bound by Narain’s argument?
(a) Narain may be granted adjournment for he is not well in the mind of the judge.
(b) Narain may not be granted adjournment for he has feigned unconsciousness.
(c) Narain may be granted adjournment for the case is a tough one and he needs to prepare.
(d) Narain may not be granted adjournment for the CPC does not apply to Commissions and thus they are not
bound by the rule of circumstances being “beyond his control”.

90. Narain always feigns unconsciousness before a judge whenever he is put in a tight spot. A case was being argued
before the Calcutta High Court. Since Narain is now very experienced at falling unconscious and seeking
adjournments, he keeps doing it whenever he finds a case difficult to deal with. As expected, this time too, the judge
believes him to be genuinely unwell.
(a) Narain may be granted adjournment for he is not well.
(b) Narain may not be granted adjournment for he has feigned unconsciousness.
(c) Narain may be granted adjournment for the case is a tough one and he needs to prepare.
(d) Narain may not be granted adjournment for such acts are against the interest of justice.

91. Narain is lazy and keeps procrastinating since the feigning adjournment business has been working for him quite
well now. Before the same judge, he has taken adjournments on 23/05/2016 and on 5/08/2016. The lawyer for the
other side, learning from Narain’s tactics also performs similar courtroom drama in the form of extreme headache
and the judge grants him adjournments on 29/08/2016 and 11/09/2016. Narain is again found to be in a tough spot
during the final arguments on 30/09/2016 and yet again seeks for adjournment. This time however, Narain is
genuinely feeling unwell because of too much partying last night.
(a) Narain may not be granted an adjournment for he’s the reason for such immense delay.
(b) Narain may be granted an adjournment for the circumstances are beyond his control.
(c) Narain may not be granted an adjournment for the matter has been adjourned for more than three times now.
(d) Narain may be granted an adjournment in the interest of justice.
92. Which of the following scenarios would be the most likely to be granted an adjournment by the court?
(a) Devgan was arguing before Justice Amish of the Trivandrum Civil Court. He sought an adjournment on the
ground that his junior was sick and there was no one to take care of the junior. Such circumstances were beyond
his control and thus fell under the relevant rule.
(b) On 29/04/2019, having argued for almost 2 hours, Goswami stated that his medical condition did not allow him
to argue for more than 2 hours before the court and thus sought for an adjournment. He has already sought
adjournments on 23/01/2019, 24/02/2019 and 15/03/2019.
(c) Rabish submitted before the court that he wanted to pick his daughter up from school and was thus constrained
to seek an adjournment before the court.
(d) None of these would be eligible for an adjournment.

93. Laxmi, an advocate appears before the Karnataka High Court. While arguing, she seeks her fourth
adjournment on a frivolous ground. The judge however grants her an adjournment. Decide
(a) The judge has acted illegally for he could only grant adjournments on reasons which are beyond Laxmi’s
control.
(b) The judge has acted illegally for he could only grant 3 adjournments to one person.
(c) The judge has acted legally for the rules above do not apply to his court.
(d) The judge has acted legally for adjournments are at his discretion.

94. Laxmi appears before the civil court in Rampur. She argues that she had to see a friend immediately in the hospital
and thus cannot argue a matter. Upon being asked whether she can send a junior to argue the matter, she stated that
since the matter is close to her heart, she would want to argue the same herself. Decide
(a) The judge can grant her an adjournment for the circumstances are beyond her control.
(b) The judge should not grant her an adjournment for the matter being close to her heart is not a
circumstance beyond her control.
(c) The judge must be sensitive to Laxmi’s friend being admitted in the hospital.
(d) The judge must order Laxmi’s junior to argue the case.

Passage (Q.95-Q.99): The definition of "private part" has to be interpreted as per the social context, a Special
Court dealing with cases pertaining to the Protection of Children from Sexual Offences (POCSO) Act said
convicting a person for committing sexual assault under Sections 354, 354A of the Indian Penal Code and Section
10 of the POCSO Act. (State of Maharashtra vs. Sahar Ali Shaikh). The designated POCSO judge MA Baraliya
had to consider whether the act of "touching the bums" of the victim by the accused would constitute an offence
under Section 7 of the POCSO Act. Clarifying that “the term private part is to be interpreted into the context
what is meant by it in our society,” the Court held that Google’s definition of not including bum in private part
may not be an acceptable interpretation as far as Indians are concerned. Section 7- “whoever, with sexual intent
touches the vagina, penis, anus or breast of the child …or does any other act with sexual intent which
involves physical contact without penetration is said to commit sexual assault.” Accused and other three
were laughing at her, when she been to buy bread. Second time when she was going still, they were laughing at
her. Past conduct of accused laughing at her and then touching her manifests that it was all with sexual intention,
to grab the chance. The Court noted that the accused had not touched the vagina, breast or anus of the victim.
However, it ruled that touching the posterior of the victim cannot be said to be without sexual intention. Sexual
intention is the state of mind, May not necessarily to be proved by direct evidence, such intention is to be inferred
from the attending circumstances of the case," the order read. While holding so, the Court sentenced the accused to
five years of rigorous imprisonment and payment of fine of Rs. 10,000.
[Source- Neha Joshi, ‘Google's definition of not including bum as 'private part' may not be acceptable in Indian
context: Mumbai Court convicts man under POCSO’, Bar and Bench,
<https://www.barandbench.com/news/litigation/term-private-part-to-be-interpreted-as-per-context-in- indian-
society-mumbai-court>, as accessed on 21st Feb. 2021]
95. Minakshi and Sheetal were best friends. One day they were playing in a park and Minakshi saw a boy playing
alone. She went to him and starts playing with him. In a meanwhile, Suraj, father of the boy, saw Sheetal playing
alone. He went to her and inquired her about her family. Taking the opportunity in favour, Suraj asked Sheetal to
touch upon his private body parts and offered her chocolates for doing so. He went on to make her feel comfortable
and grab her from back and put his fingers across her private parts without unclothing her. Minakshi saw the
incident and went to Sheetal’s mother and stated her same. FIR was lodged against Suraj under POCSO. Decide-
(a) Suraj is liable for sexual assault of Sheetal as he had touched upon her private parts and had undue physical
contact with her, though he had no such intent to do so.
(b) Suraj is not liable for sexual assault of Sheetal as though he had touched upon her private parts, he had not
unclothed her and there was no skin-to-skin contact.
(c) Suraj is liable for sexual assault of Sheetal as he had sufficient intention to have sexual contact with Sheetal
and had touched upon her private parts is material.
(d) Suraj is not liable for sexual assault of Sheetal as though he had touched upon her private parts, it’s not clear
whether its vagina, breasts or anus and definition is still unclear.

96. Payal was a young girl and pretty looking. She was the most beautiful girl in her locality. She went to shopping with
her friend Saloni and was encountered with a gang of bad boys who were stalking them from past 10 minutes. She
asked her not to follow them or else she will call the police. Boys asked her to remain silent and let them enjoy her
beauty. They all were giggling and one of them commented that he wants to sleep one night with Payal and other
with Saloni. Payal got enraged and called up police. The boy attacked Payal and snatched her phone and ran away.
Payal went to police station and complained of the gang. Decide-
(a) Sexual intention is to be inferred from the statements made by the boy and not only by the physical acts or
conduct. This can amount to sexual assault.
(b) Sexual intention can only be concluded by direct evidence and has to be proved in court of law. This cannot be
said to be sexual assault.
(c) Sexual intention can be inferred from circumstantial evidences and not merely by the act or conduct of the
accused. But it alone cannot amount to sexual assault.
(d) Sexual intention has to be inferred from the conduct of the boys and even though they have it, its a case of
mere stalking and not sexual assault.

97. Mehul and Tamana were in relationship from long time. But their families were against each other. They fixed
Tamana’s marriage with Rahul to which Tamana agreed readily. She asked Mehul to forget their long relationship
and she cannot go against her family decisions. Mehul got angry with Tamana and planned to take her from home
and get into sexual intercourse after which her family cannot deny their marriage. He asked Tamana to meet her for
the last time and take her to a hotel for dinner. He takes her to the bedroom and made her unconscious. He
forcefully had sexual intercourse with her and asked her to marry him. Tamana went to her family and stated the
incident. They filed a case against Mehul for sexual assault under IPC. The defence lawyer claimed that Mehul has
not committed alleged offence as he had used protection while having sexual intercourse and there was no physical
contact between them. Decide-
(a) The defence is correct as there was no skin to skin contact between Tamana and Mehul and the act of sexual
intercourse amount to sexual assault.
(b) The defence is incorrect as there was physical contact between Tamana and Mehul irrespective of skin to
skin contact and it amounts to sexual assault.
(c) The defence is correct as though there was sexual intention and forceful sexual intercourse, the act has to be
physical which not the case here is as Mehul used the protection.
(d) The defence is not correct as the act of Mehul of forceful sexual intercourse with Tamana amounts to much
graver offence of rape along with sexual assault.
98. Shabana, a Muslim girl of 10 years old, was made through typical practices of her religion from time long. She
asked her mother about the same to which her mother responded that Female Genital Mutilation and Nikah Halala
were long set religious practices followed by their community and thus everyone has to follow it. She claimed it to
be wrong and approached court to file complaint against Maulana who helped her family in conducting FGM
successfully. She alleged him of sexual assault as he had touched upon her private parts without her due consent
and should be punished for the same. Decide-
(a) The court will held Maulana liable for sexual assault of Shabana as he had touched upon her private parts and
hence construed physical contact with her.
(b) The court will not held Maulana liable for sexual assault of Shabana as he was only following the
customary practices of their religion and had consented her family for the same.
(c) The court will held Maulana liable for sexual assault of Shabana as there were enough direct
evidences to prove the alleged offence and religious practices is not an exception to the offence.
(d) The court will not held Maulana liable for sexual assault of Shabana as there was no sexual intention and
physical contact with private parts were part of religious practice.

99. Section 354 of IPC and other related sections covered the offences of Sexual Assault and Sexual Harassment
prescribing the punishment with gravity of offence committed. Then what is the need for implementing same
clauses with different or graver punishments under POCSO Act.
(a) Sexual Assault under IPC is different from POCSO and hence there was need to redefine the same under the
POCSO Act with higher punishment.
(b) Sexual Assault and Sexual Harassment offences are graver and heinous in cases of child and to grant
much protection to them, POCSO was implemented.
(c) POCSO Act is much more comprehensive in nature and gives detailed essentials of Sexual Assault and
Harassment then IPC and thus has overriding effect over it.
(d) IPC covers only limited number of cases of sexual assault and is gender neutral but POCSO is not gender
neutral and thus was need to implement it.

Passage (Q.100-Q.104): The incidence of income tax under the Income-tax Act, 1961 is based on the residence
of the taxpayer in a previous year, which is from 01st April to 31st March and the source(s) of his or her income.
While a person "resident" in India is taxed on his or her worldwide income, non- residents are only taxed on their
India-sourced income. Section 6 of the ITA treats an individual to be "resident" in India in any previous year if he or
she is in India for: (a) 182 days or more in that year; or (b) 60 days or more in that year and has been in India for
365 days or more in the four years preceding that year.
A writ petition was filed before the Supreme Court by a UAE-based NRI who, having come to India in March 2020,
was finally able to return to the UAE only after spending upwards of 182 days in India during FY 2020-21. The
petitioner therefore sought a direction to the effect that he would be considered "non- resident" for Indian income
tax purposes for FY 2020-21, irrespective of the duration of his stay in India, on account of the COVID-19
pandemic.
Taking note of the fact that certain relaxations had been granted by the Central Board of Direct Tax for FY 2019-
20 through the Circular, and the fact that the pandemic had continued beyond March 31, 2020, because of which
many people had remained stranded in the country, the Supreme Court directed the petitioner to make a
representation to the CBDT and further directed the CBDT to consider the same within three weeks of the receipt
thereof.
English courts have held since time immemorial that residence must be "adopted voluntarily and for settled
purposes", and Indian courts have followed suit. For instance, in CIT v. Suresh Nanda, the Delhi High Court held
that where a person was compelled to stay in India because his passport was impounded had to be excluded in
determining whether he was "resident" in India for the relevant assessment year. Delhi high court held that if the
record discloses materials that the stay (to qualify as resident Indian) lacked volition and was compelled by external
circumstances beyond the individual's control, she or he cannot be treated as a resident Indian.
Anandapadmanabhan Unnikrishnan, Tax Residency In The Time Of COVID - What Does The Supreme Court
Ruling In Gaurav Baid v. Union Of India Mean For NRIs? (Live Law, 01 March 2021)
<https://www.livelaw.in/columns/covid-19-tax-residency-supreme-court-gaurav-baid-v-union-of-india- 170505> as
accessed on 06 March 2021.

100. Dheer was an Indian citizen, who worked at TCS, Bengaluru as a consultant for a salary of INR 55,000 per month.
He worked there from 01 April 2019 to 27 September 2019. After he resigned, he applied for jobs at multinational
companies in foreign countries for better career prospects. On 01 October 2019 he left for the USA, where he had
been invited to work as chief project engineer for SpaceX for a salary of INR 1,20,000. He worked there from
1.10.2019 to 31.3.2020. His salary for October to December 2019 was credited in his USA bank account and the
salary for January to March 2020 was credited in his Bombay account directly. Decide which of his income would
be taxable in India for the previous year 2019-20.
(a) Since he is a citizen of India, all the income that he earns anywhere around the world would be taxable
in India as per the Income Tax Act.
(b) Since he is a non-resident only the income that he earned from TCS in the concerned previous year would be
taxable in India as per the Income Tax Act.
(c) Since he is a resident, all the income that he earns anywhere around the world would be taxable in India as
per the Income Tax Act.
(d) Regardless of his residential status, the income earned from TCS and the income from Google that was
deposited in a Bank account in India would be taxable in India as per Income Tax Act.

101. Which of the following is/are not a conclusion of the article:


A. Only Indian residents are liable to pay tax on their worldwide income, whereas non-residents have to pay taxes
on only income earned in India.
B. A person shall be liable to pay taxes if they satisfy the mere letter of the provisions and voluntariness of their
stay is irrelevant in the determination of their residential status.
C. Subject to factual circumstances, a person who was compelled to stay for external reasons shall not be liable
to pay taxes as a resident of the nation.
D. The Supreme Court upheld the ruling of Delhi High Court and allowed the petition by excluding the period
of involuntary stay of the petitioner in India for the purpose of determining residential status.
Choose the correct answer:
(a) II and IV
(b) I and III
(c) I, III and IV
(d) III and IV

102. Azaan was a spice trader from Afghanistan, who imports spices from India. Once, when he was in India to close a
deal, Pakistan declared a war against India. As a result, all of the Indian borders were sealed and no person was
allowed to enter or leave the country. The war lasted for 7 months and for all this time Azaan was compelled to stay
in India. After the war ended, he prepared to go back to Afghanistan, however, he received a notice to pay income
tax on all the income he has earned in the previous year as he qualifies as the resident of India. He has contended
that he was forced to stay in the country due to the ongoing war and did not volunteer to do so. The case is now
pending before a high court. Which of the following is the correct answer?
(a) Azaan shall be liable to pay income tax on his entire income earned worldwide as he has satisfied the criteria of
being an Indian resident.
(b) Azaan shall be liable to pay income tax on his entire income earned worldwide as the source of all his income is
the spice that he imports from India
(c) Azaan shall be liable to pay tax on only income earned in India as he is an Afghan citizen and the Income
Tax applies on only Indian citizens.
(d) Azaan shall be liable to pay income tax on only income earned in India as he has been involuntarily staying
in India and could not return to Afghanistan due to the war.
103. Mitchell was a lawyer from the UK, who was in India to represent a multinational automobile company. When the
Supreme Court reserved the judgment to be given after 3 months, the lawyer decided to stay to spare the cost of
travelling. The judgment was given in the favour of the multinational company and Mitchell prepared for going
back to the UK. When he filed returns to pay tax on income earned in India, he was notified that he had stayed in
India for a period of 193 days and thus was an Indian resident for the previous year, and thus shall be liable to pay
tax on all the income earned. He appealed against the notification before the Income Tax Appellate Tribunal on the
grounds that he was compelled to stay in India because the judgment was reserved for a later date. Choose the
correct option.
(a) Appeal shall be allowed as Mitchell did not volunteer to stay in India and was under compulsion to save the cost
of travelling, thus the period of his stay shall not be counted to determine his residential status.
(b) Appeal shall not be allowed as Mitchell was under no compulsion to stay in India and volunteered to stay for a
specific purpose and thus is a resident of India
(c) Appeal shall be allowed as Mitchell is just representing the company in India and payment for that case would
not be income earned in India Therefore, Mitchell shall not pay any taxes on his income in India
(d) Appeal shall not be allowed as payment received to defend a client in India amounts to income in India which is
taxable regardless of the residential status of Mitchell.

104. Anas was a student from Bangladesh, completing his bachelors and masters degree from India in 2015. In
November 2021, he decided to go back to Bangladesh to start up a tech firm of his own. His firm did very well in
the very first year and he earned enough to pay off all his education loans. In April 2022, he received a notice from
the Assessing Officer of Income Tax Department, Delhi to pay tax on all the income he earned in the previous year
2021-22. He has made representation before the authority and contended that he did not earn a single penny in India
and all his income in the relevant year has been earned in Bangladesh only. Which of the following is the correct
option?
(a) Anas shall be liable to pay income tax on all the income he earned including that in Bangladesh as in the
relevant previous year, he was a resident of India
(b) Anas shall not be liable to pay any taxes in India for the relevant previous year as he did not earn any income in
India even if he was a resident in that previous year.
(c) Anas shall be liable to pay income tax on only the income that has been earned in India, if any, in the relevant
previous year.
(d) Anas shall not be liable to pay any taxes in India as he is a non-resident and he did not earn any income in
India in the concerned previous year.
SECTION - D : LOGICAL REASONING

Directions(Q.105-Q.109): During the pandemic, the internet has been an indispensable health tool to millions.
Telehealth services have provided a safe way for patients to make appointments for COVID-19 testing and other
types of medical care. In fact, there was a 154% increase in telehealth visits during the last seven days of March
2020 compared to the same period in 2019. This was most likely due to public health mandates that required a shift
away from in-person care. Access to the internet is essential during a pandemic.
This has been particularly true as the vaccine has been rolled out. Signing up for the vaccine has predominantly
occurred online. This means that far fewer older adults from under resourced racial and ethnic minority
communities have been able to make appointments. In 2018, more than one in four Medicare beneficiaries had no
digital access at home. Those without digital access were more likely to be 85 years or older, members of racial or
ethnic minority communities and from low-income households. Over the years, medical and public health experts
have identified social factors – structural racism, a person’s neighborhood, access to fresh food, exposure to toxins,
income and education – that play a major role in health. These factors are often called the social determinants of
health. Experts consider structural racism, or racism ingrained in social, business, educational and health policy
and practice, to be one of the most damaging determinants.
To address the internet gap, we believe that policymakers must identify lack of internet access as a barrier and
protect against its effects. This could include reserving vaccines in under resourced racial and ethnic minority
communities for local residents and designating senior hours for those 65 and older.
Source: https://science.thewire.in/health/how-lack-of-internet-access-is-limiting-vaccine-availability-for- minorities/

105. Which of the following is most supported by the author’s argument?


(a) Accessibility to internet is a dispensable factor in getting access to healthcare
(b) Under resourced racial and ethnic minority communities usually have to rely on outdated modes of
communication thus, reducing avenues that provide access public healthcare
(c) Systemic racism is absolute violation of one’s right to life and life with dignity
(d) Medical service providers have been recorded to operate while being governed by racist policies

106. Which of the following can be inferred from the passage?


(a) The author is a person who wishes to bring changes to the medical policies to counter the effect of
unequitable distribution of resources
(b) The author is a person who wishes to ensure that virus vaccine is given to all without any problem or delay
(c) The author is a person who wishes to provide access to internet to every household in his country
(d) The author is a person who wishes to ensure that social factors should not be given heed to in medical
work

107. Which of the following, if true, would seriously undermine the author’s argument?
(a) Medical professionals are known to be very fair and unbiased when it comes to giving vaccines
(b) Scientists suggest that COVID-19 is bound to vanish off Earth within the next year
(c) The impact of social factors could be seen to affect medical services even before the age of internet
(d) Citizens of every country can reserve vaccines on a phone call without any problems or delays

108. The author’s statement that “Experts consider structural racism, or racism ingrained in social, business,
educational and health policy and practice, to be one of the most damaging determinants”-
(a) Forms premise of the above passage
(b) Forms conclusion of the above passage
(c) Forms assumption of the author to the above passage
(d) None of the above
109. Which of the following necessarily follows from the author’s argument?
(a) All citizens may not be given access to internet services
(b) All citizens may not be given access to vaccines for the virus
(c) None of the under resourced racial and ethnic minority communities would be able to get access to vaccines
(d) Both (A) & (B)

110. In the country of Mohenjo Daro,


(1) Cricket is bigger than Football and smaller than Tennis.
(2) Hockey is smaller than Tennis and bigger than Cricket.
(3) Football is bigger than Hockey.
If the first two statements are true, the third statement is
(a) True (b) False (c) Uncertain (d) Cannot be determined

Directions(Q.111-Q.115): As another virtual university semester unfolds — the second or even third for some
since the beginning of the pandemic — fatigue and declining satisfaction with this remote format seem to be
increasingly felt on both sides of the screen. On the one hand, there are students worried about the quality of the
courses they are taking, but above all, they are missing out on campus and community life. On the other hand, there
are teachers feeling breathlessly short of resources, who have been pushed overnight to change their practices and
run their classes from home. Beyond the purely pedagogical impacts, the issue of mental health for everyone is of
concern today.
What if one of the answers to the challenges of distance learning is to go back to basics and set up contexts that are
less “techno” and more human? So, instead of using yet another collaborative tool during a Zoom course, why not
simply create an atmosphere conducive to reflection through a warm decor, a little nature, something to watch or
music that is pleasant to listen to?
Similarly, why not open the virtual rooms earlier, or close them later, for those who want to exchange in a more
informal setting. Why not send the content in advance so as to take advantage of these so-called “synchronous
moments” to interact and inject some human warmth? Finally, it is possible to enhance non-visual stimuli to allow
students to take a break from their screens for a brief moment. The simple act of recording podcast episodes or
transmitting assessments via audio not only gives students a break for their eyes but also offers more flexibility in
when and where they can view the content. The opportunity has also come to rediscover the charms of a simple
telephone conversation, instead of another videoconference.
Source: https://theconversation.com/distance-learning-how-to-avoid-falling-into-techno-traps-153249

111. Which of the following, if true, weakens the authors claim?


(a) It is expected that school will reopen by next summer and students will be able to get back to actual class
(b) Students have become so tired with virtual classes that they are asking to open school even if it is dangerous
to do so
(c) The atmosphere in a virtual class is more conducive and productive than that of the actual classroom.
(d) The marks attained by students have reduced to some extent since the schools have closed

112. All of the following can be inferred except?


(a) Teachers are under a lot of pressure to come up with innovative techniques to engage students
(b) Students are under a lot of pressure to continue to concentrate and pay attention in virtual classes
(c) Inaccessibility to technology has become a major concern in virtual classrooms
(d) Distance learning has had a significant impact on mental health of various individuals

113. Which of the following statements will the author deem the truest?
(a) Students would enjoy a virtual class taught via the ‘Among Us’ videogame setting more than a dry recorded
audio lecture
(b) Teachers would be uncomfortable to now revert back to classroom teaching
(c) Students would fare better if they are taken back to class where teaching was more perceptive
(d) None of the above
114. Which of the following, if true, supports the author’s conclusion?
(a) 80% of the students are protesting to open up school for children
(b) 30% of the students are failing classes due to problems with their recently developed short attention span
(c) 45% of the teachers are protesting to keep the schools closed until the vaccine drive is over
(d) 73% parents are concerned their kids are getting addicted to their smartphones which were to be used to
attend classes

115. The patterns of reasoning in the above passage closely resembles the pattern of reasoning in each of the following
except –
(a) X is suggested to practice shooting arrows an extra hour every day to improve his accuracy
(b) Y is gradually increasing the number of weighted squats he does every day to achieve a higher level of fitness
(c) Q has started spending 15 minutes reading in the middle of his 8 hour writing job to increase
efficiency
(d) J has been working tirelessly for 7 days a week for this entire year

Directions: (Q.116): Each of the questions below consists of a question and two statements numbered I and II
given below it. You have to decide whether the data provided in the statements are sufficient to answer the question.
Read both the statements and give answer
(a) if the data in statement I alone are sufficient to answer the question while the data in statement II alone are
not sufficient to answer the question,
(b) if the data in statement II alone are sufficient to answer the question while the data in statement I alone are
not sufficient to answer the question.
(c) if the data either in statement I alone or in statement II alone are sufficient to answer the question.
(d) if the data even in both statements I and II together are not sufficient to answer the question.

116. Who among P, Q, R, S, T and U teaches Biology? Each person teaches one subject amongst English, Hindi,
Biology, History, Geography and Sanskrit.
I. P and U do not teach Hindi, Biology, History or Geography. Q and S do not teach Biology or
Geography. T does not teach Geography.
II. P and U teach English and Sanskrit but not necessarily respectively. Similarly, Q and S teach either Hindi or
History.

Directions(Q.117-Q.121): Certainly, much everyday writing humans now do may soon be done by machines
with artificial intelligence (AI). The predictive text commonly used by phone and email software is a form of AI
writing that countless humans use every day. According to an industry research organisation Gartner, AI and related
technology will automate production of 30% of all content found on the internet by 2022. Literacy increasingly
means and includes interacting with and critically evaluating AI. This means our children should no longer be
taught just formulaic writing. Instead, writing education should encompass skills that go beyond the capacities of
artificial intelligence.
Back at school, teachers experience pressure to teach writing for student success in narrowly defined writing tests.
But instead, the prospect of human obsolescence or “technological unemployment” needs to drive urgent
curriculum developments based on what humans are learning AI cannot do — especially in relation to creativity
and compassion. AI writing is said to have voice but no soul. Human writers, as the New Yorker’s John Seabrook
says, give “colour, personality and emotion to writing by bending the rules”. Students, therefore, need to learn the
rules and be encouraged to break them.
Students need to practise writing in which they are invested, that they care about and that they hope will effect
change in the world as well as in their genuine, known readers. This is what machines cannot do. AI is not yet as
complex as the human brain. Humans detect humour and satire. They know words can have multiple and subtle
meanings. Humans are capable of perception and insight; they can make advanced evaluative judgements about
good and bad writing.
Source: https://www.weforum.org/agenda/2021/02/ai-students-human-writing-education-artificial-
intelligence-written/
117. Which of the following is an assumption the author makes?
(a) The writing style of humans is now obsolete and requires a technological upgrade
(b) If enough information is given to an AI program, it can easily mimic the writing style of any human
(c) Students can be taught to write in a manner that is inimitable by machines
(d) Teachers need to keep up with recent developments in the field of AI

118. Which of the following, if true, supports the author’s view?


(a) On February 7, 2021, a software was able to write an article for the New York Times in the similar manner
as another human writer
(b) AI software’s are faster at writing than humans
(c) Students can be taught to write more creatively but only if they possess certain inherent traits
(d) Studies suggest that AI might take over the world if we keep on developing it as this pace

119. Which of the following can be inferred from the passage?


(a) AI software may be able to learn to write in a better manner but they can never replicate the subtleties and
innuendos
(b) Students need to be encouraged to break all the rules established by schools from long before
(c) AI is replacing human writers as its product is faster to achieve and more efficient
(d) All of the above

120. The statement “With proper coding, AI will be able to replicate and replace human writers in entirety” is
(a) Probably True (b) Definitely True (c) Probably False (d) Definitely False

121. On the basis of information provided above, what is the problem with the current system of schooling?
(a) The teachers and school authorities focus more on academic subjects rather than co-scholastic subjects
that expand the horizons of students
(b) The teachers are more concerned with students getting more marks rather than exploring their creativity
(c) The orthodox pedagogical methods are outdated as they focus on primary skill development that is easily
replicable
(d) The books that students learn from are outdated and do not provide for the recent technological upgrades
in the society, especially AI

122. Karan’s house is south facing. From the front door of his house Karan started walking and after walking 20 metres
straight he turned to his left and walked 50 metres. After this he turned to his right and walked 80 metres. Find the
distance between the point from where he finished his walk and the door of his house.
(a) 50√5 metres (b) 5√50 metres (c) 10 metres (d) 120 metres

Directions(Q.123-Q.127): Insurance regulator IRDAI has permitted dispersion of all micro-insurance items
through point-of-sales (PoS), with a point to further increment insurance entrance in the country.
The Insurance Regulatory and Development Authority of India (IRDAI) has made an extraordinary class of
insurance strategies called micro-insurance approaches to advance insurance inclusion among financially weak
areas of society. A micro-insurance strategy is a general or disaster protection strategy with a whole guaranteed of
Rs 50,000 or less.
Consenting to the recommendations of insurance organizations, IRDAI in a round has discarded the act of pre-
fixing the word 'PoS' on life, general and wellbeing items sold through PoS. The regulator said it has gotten
portrayals from insurance organizations mentioning to get rid of the prefix 'POS' in the item name.
Guarantors made portrayals to change the standards that made it mandatory for each strategy sold through the 'Point
of Sales People's to be independently recognized and pre-fixed by the name ‘POS'. IRDAI said the prerequisite was
to recognize the individual associated with the sales interaction.
The IRDAI (Protection of Policyholder’s Interest) Guidelines, 2017, under the issue to be expressed throughout
everyday life, general and health care coverage strategy, makes it mandatory to give the subtleties of the individual
engaged with the sales cycle. "By uprightness of this necessity, the need to
have the prefix 'PoS' gets excess as the insurance strategy itself will convey the subtleties of the individual selling
such an arrangement," the regulator said. The Authority, "thus, ends prerequisites" of utilizing the word 'PoS'
prefixed before the PoS item name forever, general and wellbeing items.
Also, IRDAI said it has been seen that points of interest like higher insurance infiltration, lower costs, expanded
choice to clients, which would otherwise accumulate to the policyholder by making micro- insurance items
accessible through POS channel are being lost. "Therefore, the Authority in the wake of inspecting the position, thus
permits all Micro Insurance results of Life, General and Health care coverage to be disseminated through the POS
additionally," the round said.
On the way of managing instances of wellbeing/individual mishap (Dad) arrangements where whole
guaranteed crosses the cutoff determined under the POS rules, IRDAI said the supporting substance is permitted
to perceive such approaches as being sourced by the POS and pay the charges to the POS. Source:
https://economictimes.indiatimes.com/industry/banking/finance/insure/irdai-allows-pos-to-
distribute-all-micro-insurance-products/articleshow/66014239.cms?from=mdr

123. Which among the following is right in regards to the goal of IRDAI to permit the conveyance of micro
insurance arrangements through the POS conveyance channel?
(a) The IRDAI is keen on expansion of insurance inclusion to an ever-increasing number of individuals in the
country.
(b) POS channel is the most mainstream dispersion network in the country and that is the reason the vast
majority of individuals are depending on this as it were.
(c) The IRDAI has perceived that micro insurance strategies are not well known as the typical approaches and
they ought to be discarded.
(d) Banks and insurance organizations are a lot keen on guaranteeing that the micro insurance items sold in
India are not the worldwide ones.

124. Which among the following is/are right in regards to the Micro Insurance approaches given by insurance
organizations in India?
I. Micro Insurance arrangements are just given by banks and insurance organizations since others are not
authorized by IRDAI to issue such approaches
II. Micro Insurance strategies are intended to offer inclusion to individuals who have a place with the
oppressed segment of the general public
III. Micro Insurance strategies are to such an extent that the total guaranteed in these approaches is Rs 50000 or
not exactly that.
(a) Both II and III (b) Both I and II (c) All I, II and III (d) Only II

125. Which among the following is/are right according to the data given the passage?
I. The POS channel of dispersion of insurance items will be utilized by just IRDAI and no insurance
organization will actually want to utilize it
II. The insurance organizations are primarily dependable behind the choice of the IRDAI to eliminate the prefix
'POS' from the names of the insurance items.
III. The POS channel can likewise be utilized to disseminate arrangements of insurance past the admissible furthest
reaches of inclusion.
(a) Both II and III (b) Both I and II (c) Both I and III (d) All I, II and III

126. Which among the following effectively clarifies the explanation of the new rules of the IRDAI in regards to the
withdrawal of the prefix POS in the names of the insurance strategies sold by POS?
(a) IRDAI made it mandatory to add the prefix of the approach nature in any strategy sold by the
insurance organizations in India.
(b) IRDAI isn't authorized to sell approaches online since it is the managing body of the insurance business
in the country however it likewise needs to keep a tab on the authorized mediators.
(c) The IRDAI has made it mandatory that all the approaches gave by the micro insurance specialists will be sold
through point of sales people as it were.
(d) The IRDAI currently has the data with respect to the point of sales people selling the insurance
strategies in India and it doesn't bode well to distinguish such approaches by name.
127. Which among the following is/are right in regards to the advantages that were not there before the choice of the
IRDAI permitting the micro insurance items to be sold through the POS channel?
(a) The policyholders couldn't get profited by lower and serious expenses offered by insurance organizations.
(b) The insurance organizations were not after the policyholders to take their strategies without making any sort
of customizations with respect to the insurance organizations.
(c) There were less choices accessible to the policyholders to browse the insurance items accessible on the
lookout.
(d) Both A and C

Directions(Q.128 and Q.129): Read the following passage and answer the items that follow :
A, B, C, D, E and F are cousins. No two cousins-are of the same age, but all have birthdays on the same day of the
same month. The youngest is 17 years old and the oldest E is 22 years old. F is somewhere between E and D in age.
A is older than (b) C is older than (d) A is one year older than (c)

128. Which one of the following is possible?


(a) D is 20 years old (b) F is 18 years old
(c) F is 19 years old (d) F is 20 years old

129. What is the number of logically possible orders of all six cousins in terms of increasing age? (a) 1 (b)
2 (c) 3 (d) 4

Directions(Q.130-Q.134): It’s been a busy – and controversial – year for Wonder Woman. In October 2016, the
United Nations made a curious appointment: Wonder Woman would be the global organisation’s new Ambassador
for Women’s Empowerment. But it was met with a great deal of criticism. While the fictional feminist icon has long
been a representative of strong, liberated women, her Western appearance, sexualised image and unrealistic beauty
don’t resonate with millions of young women around the world. They’re actually alienating.
This woman-directed, woman-led film tells a story of justice, of a character who fights evil forces for the greater
good. As Wonder Woman, Gal Godot overcomes the trite “damsel in distress” narrative and rescues her own damn
self. But are we being overly generous with the feminist label here? Rather than represent real women, Wonder
Woman satisfies the societal image of the ideal woman. Inhumanly strong, super sexy and bolstered by her
exceptionalism, Wonder Woman is a “walking contradiction of the competing demands placed on women’s
shoulders today”.
How many actual women or girls around the world can live up to Wonder Woman as a role model? Would we even
want them to? Also lacking in laudatory reviews of Wonder Woman is the idea of intersectionality – the
acknowledgement that women’s multiple identities (not just sex but also gender identity, race, class, sexual
orientation, religion and others) expose them to numerous forms of oppression. Why haven’t feminists noted that
the film is, quite simply, too Western and too white?
Source: https://thewire.in/film/wonder-woman-symbol-oppression

130. Which of the following necessarily follows from the author’s argument?
(a) It is imperative that we have more female superheroes
(b) It is pertinent to create more life-like female superheroes
(c) It is important to have more male superheroes which have a feministic approach
(d) It is pertinent to have more female superheroes which have a feministic approach

131. Which of the following, if true, weakens the authors claim?


(a) More representation in the superhero space has got women celebrate a breakthrough all over the world
(b) Majority of the audience members view the film as a dramatization of an average women’s life which still hold
certain fundamental empowering elements
(c) The film has been banned by Lebanon for its lead character being portrayed by an Israeli actor
(d) Both (A) & (B)
132. Which of the following can be inferred from the passage?
(a) Simply having a female lead is not enough if it fails to involve any intersectionality
(b) Gal Godot was an ideal choice to play the character of Wonder woman
(c) The movie should have been directed by a man which would have made it appeal to a larger audience
(d) Young women look up to women superheroes more than princesses in other films

133. The author’s statement “Wonder Woman is a walking contradiction of the competing demands placed on women’s
shoulders today” forms the:
(a) Premise that portrayal of wonder woman is not as feminist as it’s supposed to be
(b) Conclusion that portrayal of wonder woman is not as feminist as it’s supposed to be
(c) Premise that the character of wonder woman is not as feminist as it’s supposed to be
(d) Conclusion that the character of wonder woman is not as feminist as it’s supposed to be

134. What could be an apt heading for the passage?


(a) Wonder woman symbolises women’s oppression
(b) Wonder Woman: feminist icon or symbol of oppression?
(c) Female superheroes are a necessity
(d) Gal Gaddot – true Wonder Woman

135. In a plane, line X is perpendicular to line Y and parallel to line Z; line U is perpendicular to both lines V and W;
line X is perpendicular to line V.
Which one of the following statements is correct?
(a) Z, U and W are parallel. (b) X,V and Y are parallel.
(c) Z, V and U are all perpendicular to W. (d) Y, V and W are parallel.
SECTION - E :QUANTITATIVE TECHNIQUES

Directions (Q.136-Q.140): Answer these questions on the basis of the following information :

The owner of an Arts and Crafts shop sells a certain number of articles of 4 different varieties – Carpets
(∆), Jars ( ), Table lamps ( ) and Diwans (X).

136. Which variety of articles has the lowest average cost price?
(a) Diwans (b) Table lamps (c) Carpets (d) Jars

137. Which variety of articles has the highest average selling price?
(a) Diwans (b) Table lamps (c) Carpets (d) Jars

138. The article of which variety earned the highest profit?


(a) Diwans (b) Table lamps (c) Carpets (d) Jars

139. Which variety has the lowest average profit?


(a) Diwans (b) Table lamps (c) Carpets (d) Jars

140. For how many articles, the profit is more than 50% of the cost price? (a) 1
(b) 2 (c) 3 (d) 4
Directions (Q.141-Q.145): The histogram below shows the results of a survey conducted in a city. It shows
the number of families in that city having various daily incomes.
700

Number of Families
600 600
600
500
500
400
400 350 350 350
300 300
300 250

200

100

0
10002000300040005000600070008000900010000
Daily Income in rupees

141. How many families have daily income which is at least ₹7000? (a)
1200 (b) 850 (c) 1600 (d) 1000

142. How many families have daily income which is less than ₹4000? (a)
950 (b) 1250 (c) 1100 (d) 1450

143. How many families have daily income not less than ₹2000 but less (a) than ₹6000?
1550 (b) 1150 (c) 1250 (d) 1400

144. What percentage of the total number of families have daily income which is at least ₹6000? (a)
35% (b) 52.50% (c) 45% (d) 53.33%

145. What is the approx average daily income of families whose incomes are at least Rs.5000 and at most Rs.8000?
(a) Rs.6879 (b) Rs.6355 (c) Rs.7145 (d) Rs.6124
Directions (Q.146-Q.150): Study the following pie-chart and table carefully to answer the questions that
follow:

Total cars = 700


Distribution of cars

State-1
State-4 14%
26%

State-2
State-3 28%
32%

Table showing the ratio diesel to petrol engine cars which are distributed among four different states

State Diesel Engine Cars Petrol Engine Cars


State – 1 3 4
State – 2 5 9
State – 3 5 3
State – 4 1 1

146. What is the difference between the number of diesel engine cars in State – 2 and the number of petrol engine
cars in State-4?
(a) 159 (b) 21 (c) 28 (d) 34

147. The number of petrol engine cars in State – 3 is what percent more than the number of diesel engine cars in
state-1?
(a) 100 (b) 200 (c) 300 (d) 125

148. If 25% of diesel engine cars in State- 3 are AC and the remaining cars are non- AC. What is the number of diesel
engine cars in State-3 which are non-AC?
(a) 75 (b) 45 (c) 95 (d) 105

149. What is the difference between the total number of cars in State-3 and the number of petrol engine cars in State –
2?
(a) 106 (b) 112 (c) 102 (d) 98

150. What is the average number of petrol engine cars all the states together? (a) 86.75
(b) 89.25 (c) 89.75 (d) 86.25
USE FOR ROUGH WORK

You might also like